Discussion:Fonction zêta de Riemann

Le contenu de la page n’est pas pris en charge dans d’autres langues.
Une page de Wikipédia, l'encyclopédie libre.
Autres discussions [liste]
  • Admissibilité
  • Neutralité
  • Droit d'auteur
  • Article de qualité
  • Bon article
  • Lumière sur
  • À faire
  • Archives
  • Commons

juin 2006[modifier le code]

Autant pour moi. Je n'avais pas envisagé le prolongement analytique. Donc terminé.

Claudeh5 18 juin 2006 remarque essentielle: j'ai corrigé la SOTTISE qui consiste à dire que la fonction zeta de Riemann est holomorphe !

en effet! LeYaYa 18 juin 2006 à 15:23 (CEST)[répondre]

Claudeh5 18 juin 2006

Je trouve regrettable que l'on parle ici de la conjecture de Hilbert, qui non seulement n'est pas démontrée mais prend la place d'explications plus élémentaires oubliées ! On passe ici du coq à l'âne...

Je dois dire que je ne suis pas du tout satisfait de cet article pour de nombreuses raisons. D'une part il est sommaire, sans grand intérêt, et il y a de trop nombreux oublis pour que l'article soit intéressant, d'autre part on part dans les hypothèses d'Hilbert-Polya, sujet qui, s'il est d'actualité ne saurait convenir au lecteur qui ne connaît rien à la théorie de la fonction zeta de Riemann classique. Enfin, une rubrique fourre-tout de trés mauvais goût agrémente le tout. Je vais proposer une longue version de l'historique des liens entre la fonction zeta de Riemann et les nombres premiers. Cette version fait actuellement une vingtaine de pages "littéraires" au sens où il s'agit d'un texte (que j'ai écrit moi-même) se lisant facilement, mais trés dense (je le crois) du point de vue mathématique. Il est fait un rappel de quasiment tout, depuis Tartaglia jusqu'à aujourd'hui, en passant par Legendre, Riemann, Cahen, Bohr, ...

Vous risquez de ne pas reconnaître l'article de base. Dites moi ce que vous en pensez... Claudeh5 19 juin 2006 à 06:58 (CEST)[répondre]

Salut Claudeh5,
Si comme tu as l'air de le dire tu as des longues notes sur l'aspect historique de la fonction zéta je te suggère fortement de mettre le contenu dans un nouvel article Histoire de la fonction Zeta de Riemann et dans la section historique du présent article contente toi de faire un résumé et d'inclure en début de section la balise {{Loupe|Histoire de la fonction Zeta de Riemann}} qui dirigera le lecteur vers ton article plus approfondi
Ensuite puisque tu annonces toi-même que ton texte sera dense mathématiquement, laisse moi te rappeler que wikipédia est avant tout une oeuvre de vulgarisation. Les sujets les plus modernes ont leur place ici bien sur mais dans la mesure du possible essaye de rendre ton exposé le plus accessible possible, si nécéssaire en le cassant en sous-pages ou tu peux développer plus lentement (un exposé *très dense mathématiquement* ne serait vraisemblablement utile à personne sauf toi et quelques autres mais ce n'est pas la philososphie ici). Dans la mesure du possible également pense à rédiger ton texte de façon à créér des liens vers les articles de bases traitant des notions que tu utilises. Voilà, voilà, je suis content de voir que tu es motivé par ce sujet et surtout ne prend pas mon post comme une façon de te décourager, je cherche seulement à t'indiquer clairement dans quel esprit tu devrais rédiger tes futurs (et je l'espère nombreux!) articles :) Bien cordialement, LeYaYa 19 juin 2006 à 11:23 (CEST)[répondre]
Bonjour,
j'ai implanté l'historique et fait la moitié de l'article ...

Claudeh5 19 juin 2006 à 21:31 (CEST)[répondre]

Bonjour,
Actuellement, du point de vue du mathématicien, cet article est une caricature (au mauvais sens du terme). Claudeh5 22 septembre 2007 à 20:51 (CEST)[répondre]
Simple remarque : ayant essayé de lire la démonstration sur l'équation fonctionnelle de la fonction dzeta,on constate des incohérences.Elle est certainement tirée d'une lecture de l'article de Riemann cité,en référence externe,dans l'article sur l'Histoire de la fonction Zeta de Riemann.Mais elle semble incorrecte,car partant de l'hypothèse d'un nombre s de partie réelle supérieure à 1,elle exhibe à un certain moment la série de terme 1/n^(1-s) (qui ne peut etre convergente dans ce cas).La véritable logique,et c'est ce qu'a du faire Riemann,est de montrer d'abord qu'on peut construire une fonction dzeta prolongeant celle définie par la série en 1/n^s pour Re(s)>1,par une formule intégrale,et uniquement après,de prouver l'équation fonctionnelle qui relie dzeta(s) et dzeta(1-s).
je ne comprends pas l'objection. La démonstration ne fait jamais usage de 1/n**(1-s).Claudeh5 (d) 29 novembre 2007 à 20:11 (CET)[répondre]
La série en 1/n^(1-s) apparaît dans le paragraphe Preuve de l'équation fonctionnelle,

sous-paragraphe Expression pour Zeta,à la ligne 15,alors qu'on a déclaré être sous la condition que la partie réelle de s est supérieure à 1. Baudalbert2 le 30.11.07

effectivement, il y a un problème...Claudeh5 (d) 30 novembre 2007 à 07:56 (CET)[répondre]
Autour du calcul sur la fonction dzeta : le calcul d'une intégrale sur la fonction

(-x)^s/(e^x-1)/x,qui est une fonction multivaluée,a-t-il un sens ?Si oui,comment la valeur de la fonction est-elle choisie lorsque la fonction est multivoque ?Quelqu'un peut-il me renvoyer à un article qui traite ce problème ? Baudalbert2 le 30.11.07

La relation fonctionnelle a été démontrée par Riemann dans son mémoire de 1859. On en trouve plusieurs démonstrations dans le traité de Titschmarsh "The theory of the Riemann's zeta function", dans les deux éditions de 1951 et 1986. On trouve une démonstration dans Edwards, Ivic, Karatsuba, ... Je regarde pour CE type de démonstration.Claudeh5 1 décembre 2007 à 13:05 (CET)[répondre]
La rédaction de la démonstration (équation fonctionnelle de zeta) en résumé me pose deux

problèmes : 1)l'écriture de la série 1/n^(1-s) sous une hypothèse où elle n'est pas convergente. 2)l'utilisation d'une fonction multivaluée (-x)^s/(e^x-1)/x (du fait que (-x)^s est multivaluée).Or (-x)^s est évaluée deux fois dans le calcul de façon différente (grace à e^(i*pi*s), e^(-i*pi*s)) sans ambiguité.Il y a certainement une bonne raison à cela,mais elle mériterait un peu d'être explicitée. Baudalbert2 le 2.12.07

la fonction (-x)^s est multiforme. Il n'est donc pas étonnant qu'il y ait deux valeurs différentes : c'est TOUJOURS le cas le long de la coupure (qui ici est l'axe réel positif, le point de branchement étant 0). la seule vraie objection est la série sur 1/n^(1-s) qui n'est pas convergente.Claudeh5 3 décembre 2007 à 17:26 (CET)[répondre]

la démonstration de la relation fonctionnelle est fausse[modifier le code]

comme indiqué précédemment.

Bonjour,

C'est moi qui avais mis la démonstration de l'équation fonctionelle, et étant donné que j'ai repris pas à pas celle du livre "Riemann's zeta function" de H.M. Edwards j'ai été étonné qu'elle soit considéré comme fausse. J'accorde bien sûr le fait qu'elle manquait sans doute de rigueur, du à ma traduction plutôt médiocre et la "condensation" du tous. Mais je pense qu'en revoyant un peu le tous on pourrait en faire quelque chose de correcte. Qu'en pensez-vous? Deplus j'étais parti pour faire une page un peu plus compléte (car pour l'instant je ne la trouve pas super), mais si c'est effacé par la suite je n'en vois pas trop l'intêret.

Désolé de ne pas avoir répondu plus tôt mais je n'étais plus passé sur la page depuis quelques semaines.

Bonjour,

C'est mieux de signer ses commentaires par 4 tildes (~}. J'ai trouver la démonstartion dans la revue singularité V2N2-3 mais elle est tout aussi fausse. Titchmarsh en donne plusieurs mais pas celle-ci (heureusement). Cependant, une des démonstrations est assez proche... Je vais essayer de récrire un article sur zeta un peu plus riche et précis. Mais je ne mettrai pas de démonstration qu'on peut trouver ailleurs. Pour edwards, je vérifie.Claudeh5 (d) 8 décembre 2007 à 11:01 (CET)[répondre]

Bonjour, c'est moi qui est supprimé la démonstration, d'ailleurs après t'avoir aidé à la mettre en page. Comme je l'ai dit dans le commentaire de la suppression, la démo est toujours dans l'historique et si tu arrives à la corriger tu peux toujours t'en servir comme support. J'espère que tu n'es pas trop déçu de voir ton travail effacé, mais je pense qu'il n'est pas acceptable de laisser une démonstration dont on sait pertinament qu'elle est fausse. Cordialement, Valvino (discuter) 8 décembre 2007 à 23:02 (CET)[répondre]

RadeonXT (d) 9 décembre 2007 à 01:08 (CET) Rebonjour,[répondre]

Je viens de trouver l'historique de l'article (hier je crois que je suis toujours tombé sur l'historique de la discussion). Et grâce à toi valvino je sais aussi signer mes commentaires :-). En résumé si je comprend bien le problème principal est l'utilisation d'une suite non convergente? 1/n^1-s pour s>=0 En tous cas dans le Edwards il y a plusieurs long paragraphe entre les différentes étapes pour justifier la validité des calculs pour tous s. Je vais donc essayer de relir attentivement ces passages et voir si il y a moyen de justifier ces "incohérences" de manière rigoureuse. Maintenant que je sais où trouver les commentaires sur un article je viendrais plus souvent pour discuter des problèmes (nombreux pour des sujets aussi ardus que celui-ci).

Dans le traité d'Edwards la démonstration s'effectue bien ainsi MAIS la prémisse est autre. Il part de la représentation intégrale de zeta(s) qui est valable 'pour tout s' donc même si Re(s)<0. Il suppose ainsi Re(s) <0 fixé. Il effectue le calcul et tombe sur la série sum_1^infty(n^(s-1)). Cette série est convergente car Re(s) <0 et vaut zeta(1-s).Claudeh5 (d) 9 décembre 2007 à 08:42 (CET)[répondre]
Je ne pense pas que la démonstration de la relation fonctionnelle de zeta était "fausse"

mais qu'elle était un peu imprécise sur des hypothèses. 1)la série 1/n^(1-s) ne pouvait être écrite que si Re(s)<0 (c'était sans doute le cas dans l'esprit,mais ce devait être écrit pour que le texte devienne clair) 2)pour la fonction multivaluée (-x)^s,je crois que l'explication est la suivante:il s'agit de montrer que le résultat du calcul est indépendant de la détermination que l'on choisit de la fonction.Or,le calcul a choisi une détermination de (-x)^s,sans montrer que le résultat est indépendant de la détermination. Je crois que la démo ainsi clarifiée était correcte.Je pense donc que vous l'avez retiré un peu trop précipitamment,et que vous auriez mieux fait d'indiquer qu'il fallait revoir les hypothèses en détail. Baudalbert2 le 13.12.07

Mon intention n'était pas d'aboutir à une suppression d'une partie de cet article.Ne

pensez-vous pas qu'il est préjudiciable à la stabilité de l'encyclopédie,que des parties soient retirées des articles par des personnes qui n'en sont pas les auteurs ?Pour ma part,je voulais inciter à une correction de l'article(par son auteur),pas à la suppression d'une partie.Je pense que la suppression est toujours une perte,que mieux vaut un processus de "correction" progressive des erreurs,pour converger vers un article correct. Baudalbert2

Encore une fois, rien n'empêche de reprendre la démonstration qui est toujours dans l'historique pour la corriger et la remettre dans l'article, que ca soit par son auteur (RadeonXT ici) ou par quelqu'un d'autre (toi ou Claudeh5 par exemple) Valvino (discuter) 13 décembre 2007 à 23:09 (CET)[répondre]
Le problème est que la démonstration est difficilement récupérable: deux démonstrations d'Edwords ont été fondues en une seule. Or les prémisses de la première sont différentes de celles de la seconde. L'erreur vient de là. Mais c'est tellement fondu qu'il faudrait tout récrire !Claudeh5 (d) 14 décembre 2007 à 14:03 (CET)[répondre]
Je pose la question suivante:est-ce que la démonstration serait consultable en ligne dans une référence externe de l'article? (apparemment non,car les références citées ne sont pas consultables).Seul le travail original (de Riemann) est accessible sur un site anglais (référencé je crois dans l'article Sur le nombre de nombres premiers inférieurs à une taille donnée);mais il est plutôt elliptique,et écrit dans un langage littéraire qui est quasiment irrecevable aujourd'hui.Donc il n'y a pas moyen de consulter une démonstration en ligne(?)On nous dit qu'une fonction a été définie pour tous les complexes.Mais comment ?par quelle formule ?On établit une relation fonctionnelle pour une fonction qui n'a pas été définie.Où est cette fonction mystère ?Baudalbert2 le 21.12.07

Suggestions sur l'article[modifier le code]

Bonsoir, j'ai rajouté la condition de convergence oubliée pour le produit eulérien. Sinon, c'est un article très dense, très complet, il me semble.

  • Il manque amha une introduction expliquant l'intérêt de cette fonction, son rôle, etc.. Il me semble qu'il faut plus mettre en avant l'hypothèse de Riemann qui reste une des grandes conjectures indémontrées en maths.
  • Personnellement, en liaison avec la discussion qui précède, j'aime bien la preuve donnée par Cartier dans (en) M. Waldschmidt, P. Moussa, J.-M. Luck et C. Itzykson (éds.), From Number Theory to Physics [détail des éditions]

pour le prolongement à C-1: on a des formules, pour chaque entier n, exprimant la fonction zeta (pour Re(s)>1)comme 1/(s-1)+ des termes polynomiaux en s + une intégrale de 1 à l'infini avec comme intégrant un terme en . C'est une application des formules de sommation d'Euler-Mac Laurin. Le terme en a l'intérêt d'être périodique de période 1 donc reste borné, donc l'intégrale converge en fait dès que Re(s)>1-n (donc prolonge zeta là-dessus). On montre que sur les parties en commun les fonctions sont égales et ceci permet donc de définir la fonction zeta sur tout C. L'intérêt de cette démonstration, à mon avis, c'est qu'on peut facilement résumer la preuve pour expliquer comment le prolongement se fait, de manière assez élémentaire et concrète (il ne faut que deux ou trois lignes de plus que ce que je viens d'expliquer, pour expliquer les fonctions et donner exactement la formule les liant à zeta). Ensuite on peut juste indiquer quelques autres types de démonstrations comme celle avec un contour de Hankel (si j'ai bien compris, celle qui avait été donnée avant). J'écris tout cela volontiers si j'ai le feu vert.

  • Pour le reste de l'article qui me semble actuellement assez technique, j'aimerais bien qu'on explique au fur et à mesure pourquoi on fait tout cela, etc. Je vois pour ma part les études sur les fonctions zeta, L, comme suivant un schéma assez standard (produit eulérien, prolongement analytique, étude des pôles et des zéros, valeurs spéciales, applications), sauf que bien sûr certaines parties dépveloppent une vie propre, mais je me demande si cela ne clarifierait pas les choses pour un lecteur ne les connaissant pas du tout d'expliquer cela un peu (ou bien, on fait un article introductif sur toutes les fonctions de ce type et on renvoie aux articles plus spécialisés comme celui-ci ?). Toutes mes amitiés, dites-moi si vous pensez que je dois rédiger la preuve ci-dessus. --Cgolds (d) 6 janvier 2008 à 01:38 (CET)[répondre]
Oui, tout a fait. Je connais le prolongement par les polynomes de Bernoulli. Je parlerai aussi à un moment de celui qu'on obtient en prenant le série de dirichlet alternée.Claudeh5 (d) 6 janvier 2008 à 09:05 (CET)[répondre]
✔️ pour les Bernoulli. Merci de regarder si cela convient, et merci aussi pour l'intro. C'est une bonne idée de parler de Dirichlet alternée et du prolongement sur Re(s)>0 ! Bien cordialement, --Cgolds (d) 6 janvier 2008 à 16:44 (CET)[répondre]

Je préviens tout de suite que je ne suis pas spécialiste du domaine, donc je ne toucherais à rien, mais ne pourrait-on pas s'inspirer de l'article anglais et mettre quelques illustrations? Skiff (d) 6 janvier 2008 à 16:55 (CET)[répondre]

Ce qui vient de se passer sur le bouleversement de l'article "fonction zeta de Riemann" illustre les défauts majeurs de l'encyclopédie,surtout son extrême instabilité.Effacer tout le contenu d'un article,pour produire carrément quelque chose de tout à fait différent,ce n'est pas une façon de faire,surtout si on se réfère à l'objet d'une encyclopédie,avant tout pédagogique.Un texte doit permettre sa discussion par un non spécialiste,sinon nous sommes plutôt dans quleque chose qui ressemble à une revue de chercheur.C'est l'impression que donne désormais pour moi cet article.La rédaction du précédent contenait peut-être des erreurs (les erreurs sont souvent le plus intéressant pour progresser),mais elle était encore accessible au béotien.Comme je l'avais déja indiqué,je pense qu'il est de très mauvaise politique de chambarder un article qui a été écrit,plutôt que d'essayer de l'amender.Que ,si on veut produire quelque chose de totalement différent,autant vaut de créer une nouvelle référence et un nouvel article.Baudalbert2 le 7.01.2008
Je dois dire que je ne comprends abasolument pas votre reproche: l'article initial (qui existe encore dans l'historique) était d'une indigence à faire pleurer. La démonstration de la relation fonctionnelle (que manifestement vous réclamez) ne fait pas partie des nécessités d'une encyclopédie qui ne peut et ne doit en fin de compte que résumer les connaissances sur un sujet donné. Vous avez de plus demandé que l'on vous explique comment la série de dirichlet de zéta se prolongeait à C-{1}. On vous présente en fait deux démonstrations, l'une par les polynômes de Bernoulli et la formule d'euler-maclaurin, l'autre par la série de dirichlet alternée (complétée par la relation fonctionnelle). D'autre part cette version de l'article donne beaucoup plus d'informations que la précédente. Mais il est clair qu'une encyclopédie, même en ligne, ne remplacera jamais un traité sur le sujet en question. La documentation dont je dispose sur cette fonction est constituée de milliers de pages (avec il est vrai des redites). Le nombre des articles dépasse les 3000, et beaucoup me sont inconnus. J'ai mis les éléments essentiels de la théorie. L'étude de cette fonction n'est pas du niveau du béotien. La démonstration de la moindre propriété occuperait plusieurs pages et souvent il faut raccourcir sans donner toutes les explications. j'ai ainsi travaillé sur un article de Motohashi qui faisait initialement 5 pages mais qui atteint facilement les 20 pages quand on a mis tous les détails passés sous silence. Et encore, cela n'est probablement pas compréhensible par un étudiant de licence de math.Claudeh5 (d) 7 janvier 2008 à 18:14 (CET)[répondre]
Je crois que le reproche ne vient pas tant du contenu que de l'absence d'explications sur les transitions, etc. Mais ce n'est pas facile de travailler en collaboration et peut-être un peu de patience est requise, svp, Baudalbert2. Claudeh5 a déjà commencé à mettre une introduction, et je suis sûre que les autres transitions et explications viendront, plusieurs personnes sont prêtes à l'aider si nécessaire. Une solution aurait été de fabriquer dans un coin un article avec toutes les qualités (complet, lisible à plusieurs niveaux, début accessible à tout le monde dans un style journalistique, etc.) avant de le mettre ici, mais personnellement je ne crois pas que ce soit la bonne méthode, d'une part parce qu'alors le moindre changement crée de terribles tensions, d'autre part parce que le but de WP est d'écrire selon ce qu'on sait faire, et de profiter ainsi de la collaboration, etc. Vous nous avez demandé de définir la fonction sur tout C -1: êtes-vous maintenant satisfait de cette partie, souhaitez-vous d'autres explications là-dessus ? Si vous pouviez nous dire ce que vous ne comprenez pas sur ce début, cela permettrait d'avancer. Par exemple, Claudeh5 (et moi-même aussi) tablons sur les liens, nous ne reexpliquons pas ce qu'est un prolongement analytique pour une fonction méromorphe d'une variable complexe, est-ce que cela vous pose un problème ? Qu'est-ce vous aimeriez trouver sur le début qui n'est pas là ? Merci de préciser vos problèmes sur cette petite portion afin d'essayer d'améliorer l'article. Cordialement, --Cgolds (d) 8 janvier 2008 à 00:20 (CET)[répondre]
On ne peut que se féliciter que l'extension de la fonction ait été donnée dans l'article, en montrant sa définition par une formule intégrale.Cependant,il reste un doute sur ce qui avait été affirmé précédemment,comme quoi Riemann avait démontré qu'on pouvait construire l'extension,ce qui ne paraît pas franchement évident quand on consulte son mémoire.
La critique sur le changement de l'article était d'une autre nature.Elle s'élevait contre le changement de fond en comble de l'article,provoquant une impression de grande instabilité,pour le non professionnel qui recherche des points de repère et de progression éventuels,qui préfère donc se trouver face à un texte stable,et dont les lacunes (ou des erreurs éventuelles) sont comblées par de légers ajouts ou corrections.Dans une encyclopédie "coopérative",seul le premier auteur d'un article a un droit absolu,et il faut être extrêmement réservé sur la façon de le corriger (sauf s'il s'agit d'erreurs manifestes).
l'article initial comportait de très nombreuses lacunes. J'avais déjà initialement corrigé la sottise dans l'affirmation que la fonction était holomorphe. Il y avait d'autre part e manifestes manques et une démonstration fausse de la relation fonctionnelle. Il reste beaucoup de choses à dire: les estimations à partir des zéros, les implications de l'hypothèse de Riemann, de l'hypothèse de Lindelöf, de celles de Mertens. Je parlerai peut-être du phénomène de prolifération des zéros... J'ajoute qu'il ne saurait être question de laisser un article indigent. J'ai essayé de garder le plus possible de l'ancien article. Mais il est vrai qu'il a beaucoup changé. Il sera complété, notamment sur les critères équivalents à l'hypothèse de Riemann. Maintenant, si vous préferez l'ancien...Claudeh5 (d) 24 janvier 2008 à 23:00 (CET)[répondre]
concernant l'article de Riemann, celui-ci contient beaucoup de chose et, comme d'habitude chez Riemann, les démonstrations et justifications sont soient absentes soient ébauchées. Je rappelle que sur les cinq conjectures de l'articles, quatre sont démontrées par Hadamard et Von Mangold entre 1892 et 1903. Il reste l'hypothèse de Riemann. Son explication concerant la relation fonctionnelle est parsemée de nombreux trous mais n'est pas fausse (seulement imcomplète). Certaines de ses justifications sont carrément fausses vers la fin (Li(x) - pi(x) changent de signes une infinité de fois, l'estimation n'est pas en racine(x) mais en racine(x) ln x ...Claudeh5 (d) 24 janvier 2008 à 23:09 (CET)[répondre]
Précisons que le contributeur initial (COLETTE) a quitté wikipedia en 2004.Claudeh5 (d) 24 janvier 2008 à 23:39 (CET)[répondre]
Pour continuer sur ce sujet, il est raisonnable d'avoir une certaine stabilité quand l'article a atteint un stade avancé (ex :bon article ou excellent article), mais les ébauches etc. ont vocation à être transformées sérieusement. Heureusement, vu le travail de Claudeh5, celui-ci va certainement rester stable bientôt pour un bon bout de temps ! --Cgolds (d) 1 février 2008 à 19:29 (CET)[répondre]

Etat actuel[modifier le code]

Opinion de Cgold[modifier le code]

Le prologue me plaît bien ! Deux détails : un bout de phrase qui traîne (définitions, etc.) à intégrer ou enlever je n'étais pas sûre. A propos des hypothèses plus faibles, tu ajoutes 'sans succès', c'est vrai pour certaines, mais certains résultats sur la localisation des zéros sont prouvés, donc j'enlèverais le 'sans succès', trop général.

Un dernier point : je ne sais pas s'il faut regretter l'absence de preuves. C'est une encyclopédie après tout, pas un manuel. Peut-être si tu le veux vraiment, dire simplement : 'Les recherches sur la fonction zeta constituent un domaine très technique. La plupart des preuves, nécessitant une formation spécialisée en théorie analytique des nombres, sont omises ici.' (compte tenu des discussions sur wp, pour beaucoup, 'avancé en maths' veut dire licence de maths par exemple, ce qui, amha, ne suffirait pas pour les preuves les plus délicates dans ce sujet). Bravo ! --Cgolds (d) 1 février 2008 à 19:25 (CET)[répondre]

Opinion de jl[modifier le code]

Je partage dans les grandes lignes les idées de Cgolds. Manifestement comme elle, j'ai pris beaucoup de plaisir à lire l'article. Des pistes pour l'améliorer encore, j'en propose une mais je ne suis pas très sur de moi (l'avis doit donc être pris avec des pincettes).

Je me demande si le prologue ne peut pas toucher un public un peu plus vaste qu'à l'heure actuelle. Des livres grands publics à tirages finalement pas si confidentiels me laisse penser ainsi.

  1. Pourquoi ne pas indiquer que la fonction zêta est une fonction complexe à valeurs complexes en des termes plus simples ?
  2. Pourquoi ne pas indiquer qu'elle possède de bonnes propriétés de régularité (c'est à dire qu'elle est lisse partout où elle est définie, ce que l'on appelle méromorphe) ?
  3. Pourquoi ne pas indiquer qu'un zéro est un point ou l'image de la fonction vaut zéro ?
  4. Pourquoi ne pas indiquer que la connaissance de l'emplacement exact des zéros qui ont une partie imaginaire non nulle donne la répartition exacte des nombres premiers ?

Ensuite, de toute manière c'est un peu mission impossible d'aller plus loin pour le grand public. L'utilisation d'un langage plus métaphorique perturberait les experts sans pour autant éclairer les néophytes, le choix de Claudeh5 me semble donc pertinent.

Enfin, je regrette sans illusion la présence des démonstrations. Il suffit de lire l'article Fonction méromorphe pour se rendre compte qu'ajouter maintenant les preuves ne fait pas sens. J'espère que dans cinq ans, introduire à l'aide d'une multitude d'articles connexes le savoir purement mathématiques sera possible. Le choix de Claudeh5 est pour moi, hélas, le bon.

Pour finir merci encore pour ce bien bel article, je pense qu'il remplit en large part sa fonction pour la majorité des lecteurs potentiels. Jean-Luc W (d) 15 février 2008 à 11:06 (CET)[répondre]

Début de relecture[modifier le code]

On s'achemine (enfin, Claudeh5 nous achemine ) vers un article très complet. Je n'ai pas encore tout relu, je vais essayer de le faire dans les jours à venir.

Je crois qu'il y a encore quelque chose à lisser dans le prologue (désolée, désolée...). Probablement dans le sens de Jean-Luc W, car pour l'instant je trouve que l'hypothèse de Riemann (dont il n'est d'ailleurs pas dit ici qu'elle concerne la fonction zeta et pas toutes les fonctions méromorphes) a l'air d'occuper tout l'espace, c'est un peu perturbant (c'est un problème de rédaction,pas de fond, bien sûr).

Ensuite, je ne comprends pas bien ce qui est dit sur la série de Dirichlet juste après. Est-ce que tu annonces à l'avance le prolongement analytique qui sera établi plus loin , Parce que là on a l'impression que c'est une donnée évidente (se prolonge à tout le plan sauf 1, etc.).

Dernier point, moins important : les valeurs spéciales de fonctions zeta ou L sont devenues très importantes en géométrie arithmétique (à des périodes près comme pi, les rationnels mis en évidence témoignent de structures), donc ce serait bien de remonter ce paragraphe plus haut, d'autant que c'est une partie bien compréhensible.

Je vais faire des propositions plus effectives après relecture globale, c'est plus facile quand on lit de manière extérieure. Les illustrations sont aussi très intéressantes. A suivre !--Cgolds (d) 18 février 2008 à 23:29 (CET)[répondre]

j'ai l'intention de mettre le tableau de valeurs et l'image "rouge" (qui est génante) côte à côte. Il y a encore probablement à regrouper les estimations dans les différentes régions mais cela va nécessiter une petite refonte d'un paragraphe. Le problème des moments n'est pas encore traité. Je n'ai pas non plus traité des valeurs de la fonction. il y a ici quelques énoncés amusants. Il faudrait que je parle aussi des applications ! j'ai prévu un petit paragraphe sur la fonction M(x) qui s'exprime théoriquement avec les zéros. Il y a d'autres fonctions courantes qui sont liés à zeta de la même manière.Mais tout cela n'est pas censé bousculer l'article.Claudeh5 (d) 19 février 2008 à 01:05 (CET)[répondre]
J'ai enlevé pour l'instant deux lignes dans le paragraphe sur la définition par la série de Dirichlet, car sans doute à la suite d'un copier-coller, elles disaient qu'il y avait un prolongement partout sauf en 1 comme si la série même convergeait évidemment. Il me semble qu'il y a aussi un problème implicite de prolongement dans le paragraphe suivant sur le log (car on parle des valeurs en -2 etc., comme si on savait déjà ce que c'est). Soit il faut déplacer après le prolongement, soit il faut indiquer explicitement qu'on peut prolonger le log. Amicalement, --Cgolds (d) 20 février 2008 à 10:43 (CET)[répondre]

Les zéros triviaux[modifier le code]

Désolé si ma question est stupide, mais je la pose quand même; cet article (ainsi que d'autres ouvrages consultés) disent que les zéros triviaux de la fonction sont en -2, -4, -6... Or, même en réfléchissant intensément, pour moi,

.

et non 0. Non?Thémistocle (d) 8 décembre 2008 à 22:55 (CET)[répondre]

Ce que tu n'écris n'est pas faux dans l'absolu, mais ça n'a pas de sens! La fonction est tout d'abord définie par une série pour tout complexe dont la partie réelle est >1! Ensuite on l'étend à tout le plan complexe, mais les valeurs ne sont plus donnée par cette série, car elle n'est plus convergente quand la partie réelle est <1. Elle sont par exemple données par l'équation fonctionnelle. Valvino (discuter) 8 décembre 2008 à 23:14 (CET)[répondre]
OK, merci beaucoup, j'ai compris.Thémistocle (d) 8 décembre 2008 à 23:37 (CET)[répondre]

Deux "défauts de matheux"[modifier le code]

Dans l'objectif d'un label, je pense que l'article devrait se mieux conformer aux usages de wikipedia :

  • Sourcer les affirmations, les formules, etc afin que le lecteur qui voudrait vérifier ou aller plus loin sache exactement où chercher (livre de référence + renvoi à la page ou au chapitre concerné)
  • Développer l'historique : une petite synthèse de histoire de la fonction zêta de Riemann. Si cette synthèse est bien faite, elle doit en même temps servir de "deuxième prologue" et donc plutôt se trouver au début de l'article. ---- El Caro bla 21 mars 2009 à 15:01 (CET)[répondre]
(conflit d'edit) A mon avis, en me basant sur mes souvenirs et en ayant juste parcouru rapidement cet article, il souffre de gros défauts : on dirait un formulaire qui aurait été constitué en pompant tout et n'importe quoi dans la littérature - même si le tout est en fait certainement vrai et certainement intéressant, rien n'est fait pour le montrer au lecteur : aucun travail d'organisation ou de rédaction. Il y a de beaux ouvrages formulaires en maths, il me vient à l'esprit Whittaker et Watson, que je n'ai pas sous la main, mais j'imagine que les auteurs faisaient quand même un effort minimal pour dire où ils voulaient en venir.

Je ne vais pas tout critiquer point par point, mais deux exemples :

  • dans le partie sur le log de la fonction zeta : on décrit des coupures à faire pour définir ce log. Quel intérêt ? Pourquoi le faire comme ça ? Soit le lecteur sait qu'on est en train de définir une surface de Riemann sur laquelle le log sera univoque (mais on ne prend même pas la peine en définitive de préciser la surface obtenue, hic), et il se demande bien où on veut en venir, soit il ne le sait pas, et il se demande encore plus où on veut en venir !
  • Pourquoi consacrer une section entière à la valeur de la dérivée aux zéros triviaux ?

Evidemment, s'il faut, je pourrais détailler sur les sections ; mais il faudrait vraiment commencer par établir un plan. Cordialement, Salle (d) 21 mars 2009 à 15:09 (CET)[répondre]

question 1[modifier le code]

Je pense qu'il y a des fautes d'écriture dans Euler-MacLaurin (confusion entre n et N) dans ,, et .

il y avait une faute de N écrite n dans le reste R au début. La confusion vient probablement du fait que l'on utilise N au début et n dans la formule pour zeta. On pourrait effectivement remplacer les N en n pour clarifier.Claudeh5 (d) 18 octobre 2009 à 09:51 (CEST)[répondre]

question 2[modifier le code]

Je voudrais avoir un éclaircissement sur les zéros non triviaux de la fonction .Vous définissez dans le paragraphe sur ces zéros la fonction et la fonction.Est-ce que vous considérez s comme réelle dans cette définition?Alors vous appliquez un théorème sur les zeros des fonctions entières (Picard?).Ce théorème est référencé dans les fonctions entières pour des fonctions de variable complexe.Donc,si s est réel,cela signifie que vous appliquez un théorème sur l'infinité des zeros, valable si la variable est simplement réelle.Est-ce précisément le cas ? Il n'est pas dit explicitement que s est réel ici.--90.15.206.220 (d) 25 octobre 2009 à 23:20 (CET)[répondre]

Non, la fonction est considérée comme une fonction de la variable complexe s. D'autre part, ce n'est pas le théorème de Picard qui est appliqué mais le fait que la fonction est d'ordre 1 et donc que , qui est paire (c'est essentielle) est d'ordre 1/2. Ce seul fait entraine qu'elle admet une infinité de zéros complexes (voir fonction entière). Par suite de la symétrie, les zéros complexes se traduisent par des zéros complexes de dans la bande [0,1]. On sait d'autre part qu'il y a une infinité de zéros réels de (théorème de Hardy, 1914) mais peut-être pas tous. Si tous les zéros sont réels, alors l'hypothèse de Riemann est vraie mais pour l'instant cela reste une conjecture.Cela a-t-il répondu à votre interrogation ?Claude le pénible (d) 26 octobre 2009 à 07:18 (CET)[répondre]
Merci pour cet éclaircissement.En fait j'ai été conduit à penser inconsciemment que s était supposé dans ce paragraphe, dans R à cause de l'écriture : .--90.15.79.94 (d) 28 octobre 2009 à 18:32 (CET)[répondre]
Pour eviter cela, je vais le préciser.Claude le pénible (d) 28 octobre 2009 à 20:07 (CET)[répondre]

Bonjour, Je trouve que dans cette partie de l'article (je n'ai pas encore tout vu) vous mélangez les variables s, t et σ et ça devient vraiment "pénible" monsieur claude. je ne vais pas corriger à votre place, je vous laisse prendre le soin de le faire pour ce paragraphe des zéros non triviaux ainsi que partout dans les paragraphes qui sont en lien avec celui-ci dans l'article . En vous remerciant --Mourad ZAMOURI (discuter) 17 avril 2019 à 10:39 (CEST)[répondre]

question 3[modifier le code]

Y a-t-il un bug dans les premières écritures du paragraphe de la fonction S(t)? Ne faut-il pas considérer les variables t et T comme les mêmes dans les premières expressions de N(T) et S(T) (si je ne me trompe pas) ?

Absolument exact. Il n'y a que des T. J'ai corrigé. Merci de votre vigilance.Claude le pénible (d) 3 novembre 2009 à 13:33 (CET)[répondre]

question 4[modifier le code]

Dans le paragraphe sur les zéros non triviaux,un point me semble difficile à comprendre.Quand vous dites que c'est par une intégration de qu'on évalue la différence entre le nombre de poles et le nombre de zéros,je ne vois pas comment vous faites le lien.Pouvez-vous développer quelques détails,ou préciser cette intégrale ? Merci,si ce n'est pas trop long à faire.--90.0.64.244 (d) 4 novembre 2009 à 08:40 (CET)[répondre]

C'est pourtant une formule classique de la théorie des fonctions de la variable complexe. Soit donc f(z) une fonction analytique complexe dans un domaine D borné ayant des zéros et des pôles dans le domaine mais pas sur sa frontière (condition facile à satisfaire en déformant au besoin un peu le domaine. Soit m la multiplicité d'un zéro a(qui est isolé). Si l'on calcule f'(z)/f(z) au voisinage de a, on trouve f'(z)/f(z)=m/(z-a)+g(z) où g(z) est une fonction holomorphe sur un disque de centre a et de rayon r >0. Si a était un pôle d'ordre n, on aurait f'(z)/f(z) = -n/(z-a)+h(z), h(z) comme g(z). Pour le principe. Maintenant passons à D tout entier. Dans D, on applique le théorème de Factorisation de Hadamard (voir théorème de factorisation de Weierstrass). L'application du logarithme au produit de Hadamard et sa dérivation (dont la dérivée vaut f'(z)/f(z)) donne immédiatement:

où les ensembles zeros et poles ne concernent que ceux de D et où g(z) est une fonction analytique n'ayant pas de pôle dans D (elle peut en avoir ailleurs). Intégrons maintenant f'(z)/f(z) sur la frontière de D. On trouve d'après la formule des résidus que l'intégrale de g(z) est nulle (pas de pôle dans D). D'autre part, le résidu pour a dans zero est m(a)= multiplicité du zéro a. Celui de a dans pole est -n(a)= multiplicité du pole a. On trouve donc comme intégrale 2i pi (N-P), où N est le nombre des zéros dans D comptés avec leurs multiplicités, N étant de même pour les pôles. Tout cela n'est pas spécifique à zeta mais fonctionne avec toute fonction analytique complexe. Il reste à estimer zeta'(s) / zeta(s) sur les côtés de la bande critique de 0 jusqu'à une hauteur T. Là, c'est délicat et nécessite du travail...Claude le pénible (d) 4 novembre 2009 à 09:49 (CET)[répondre]

Dans la suite, une autre méthode de calcul est suggérée: la méthode basée sur le principe de l'argument.Claude le pénible (d) 5 novembre 2009 à 10:17 (CET)[répondre]
Le but de mon interrogation était en fait l'évaluation de la fonction dans ce cas pour que la méthode décrite puisse être appliquée.Je pense que pour le lecteur qui veut poursuivre l'étude de l'évaluation du nombre de zéros telle qu'elle est faite dans la suite de l'article,il est nécessaire qu'il ait une précision sur l'expression de .Or je n'ai vu nulle part ailleurs dans l'article d'expression (sauf peut-être celle avec la fonction de Von Mangolt,mais je ne pense pas que ce soit celle qu'on peut utiliser).Est-il possible que vous donniez dans ce paragraphe l'expression qui sera utilisée de dans l'application du théorème des résidus?--90.15.73.189 (d) 9 novembre 2009 à 09:40 (CET)[répondre]
La démonstration la plus simple est la suivante: (Titschmarsh, the zeta function of Riemann, 1930)

On part de pour laquelle on sait qu'on a d'après la relation fonctionnelle . Le nombre de zéros de est le même que celui de dans le rectangle défini par les sommets opposés (0,0) et (1,T), soit N(T). Si T n'es pas l'ordonnée d'un zéro, est égal à la variation de l'argument de le long du rectangle. Or, est réelle pour t=0 et également pour , de sorte que la variation totale autour du rectangle est 2 fois la variation autour de la moitié en partant de s=2. Donc est égal la variation d'argument entre 2 et 2+iT et de 2+iT à 1/2+iT le long des droites. Utilisant la formule de Stirling complexe, on a

. Il reste à montrer que le dernier terme est O(ln T).

La partie réelle de ne s'annule pas sur l'axe puisque . donc la variation de l'argument de entre 2 et 2+iT est inférieure à .

Si s'annule q fois entre 2+iT et 1/2+iT, cet intervalle est divisé en q+1 parties à travers lesquelles ne prend qu'un signe, soit + soit -. Donc dans chaque partie la variation de l'argument de n'excède pas . et ainsi la variation totale de l'argument est inférieure à . il reste à évaluer q.

Or q est le nombre de zéros de pour , et donc où n est le nombre de zéros de f(z) dans |z-2| <= 3/2. On applique alors la formule de Jensen (voir [[1]]) qui donne où 3/2 < r < 2 et parce que , le membre de droite est inférieur à A ln T.

Autrement, on trouve dans le livre de Ivic une démonstration plus longue mais plus proche de la formule indiquée qui part de la fonction et calcule . On trouve donc le terme en qui donne pareillement par Stirling les premiers termes du développement. Après, il faut estimer . Pour cela on part de la formule de factorisation de Hadamard

dont on prend le logarithme et qu'on dérive. On utilise alors certaines estimations (que je n'ai pas données ... sur certaines sommes sur les zéros) et la formule

que j'ai donnée pour montrer que .Claude le pénible (d) 9 novembre 2009 à 14:30 (CET)[répondre]

Merci pour avoir mis la démonstration pratiquement complète du calcul de N(T),cela donne du grain à moudre.--90.14.156.187 (d) 23 novembre 2009 à 17:31 (CET)[répondre]

Comme le sujet m'intéresse toujours,je me permets de continuer à vous poser des questions.

Questions 5 et 6[modifier le code]

Pour les zéros calculés sur l'axe 1/2,y a-t-il des résultats sur leur multiplicité,plus fins que la majoration que vous donnez ?

très légèrement. Voir histoire de la fonction zêta de Riemann paragraphe "De la simplicité des zéros".Claudeh5 (d) 24 novembre 2009 à 00:40 (CET)[répondre]

Pour une certaine valeur de T,est-ce qu'on sait prouver (je veux dire numériquement) qu'il n'y a pas de zéro hors de l'axe 1/2,dans le rectangle [0;1+iT]?Si oui,pour quelle valeur de T ?

Brent indiqua en 1979 l'avoir fait pour les 75 000 001 premiers zéros de zeta soit jusqu'à T=32 785 736.4 et les a trouvé sur 1/2.
Van Lune, Te Riele et Winter en 1986 ont calculé les 1,5 premiers milliards de zéros et les ont trouvés de parties réelles 1/2. Donc jusqu'à T=5 10^8. On est allé encore plus loin: les 100 premiers milliards)(Wedeniwski). Voir http://www.zetagrid.net/zeta/math/zeta.result.100billion.zeros.html pour plus de précision.Claudeh5 (d) 24 novembre 2009 à 00:40 (CET)[répondre]

Si des réponses,ou des liens, figurent dans l'article sans que je les aie remarqués,veuillez m'en excuser,c'est que je n'aurai pas lu assez attentivement.--90.14.156.187 (d) 23 novembre 2009 à 17:31 (CET)[répondre]

Mais surtout, il faut préciser que ce sont des démonstrations, et que le résultat est exact : ainsi, Riemann avait déjà démontré que le 1er zéro est en 7,... i (je crois) +1/2 et pas "plus 1/2 à 10^-12 près" : c'est bien un calcul numérique approché qui est le point de départ, mais il suffit de démontrer (par calcul numérique) qu'une certaine intégrale est inférieure à 1 pour prouver qu'elle est nulle (d'après le théorème des résidus), ce qui entraîne la conclusion --Dfeldmann (d) 24 novembre 2009 à 11:55 (CET)[répondre]
Je crois que tu ne comprends pas tout le problème. Il ne s'agit pas de montrer qu'une intégrale est plus grande ou plus petit que 1, qui ne peut qu'indiquer qu'il y a un pôle dans un rectangle mais de démontrer que la valeur est exactement 1/2. Ce n'est pas un problème de décomptage de zéros. Or le calcul numérique ne permet pas de le faire avec une précision infinie. IL S'AGIT D'UN PROBLÈME SANS SOLUTION THÉORIQUE INFORMATIQUEMENT PARLANT (je t'invite à regarder la discussion sur la méthode de dichotomie).Claudeh5 (d) 24 novembre 2009 à 13:34 (CET)[répondre]
Ah vraiment ? Tu veux dire qu'on ne sait même pas démontrer que le premier zéro est de partie réelle 1/2 exactement ? Allons ... Bon, je t'invite à aller lire ceci : Numerical calculations (en) où tu pourra voir que, essentiellement, on détermine les zéros sur la ligne critique par l'étude d'une fonction réelle sur cette ligne, puis on compte les zéros dans un rectangle de la bande critique autour de ceux trouvés (en appliquant habilement le théorème des résidus), et on vérifie qu'on les a tous. Bilan, si, si, c'est possible (voir par exemple la formule de Riemann-Siegel), et ton impossibilité théorique (en MAJUSCULES??? pas besoin de crier comme ça) m'inquiète fort...--Dfeldmann (d) 24 novembre 2009 à 21:14 (CET)[répondre]
Tu m'excusera~s mais moi je lis ceci dans le papier de Xavier Gourdon (2004):

« Our result of the RH verification until the 1013-th zero should permit to improve such quantitative estimates a little more. It is of importance here to state that numerical verification of the RH is proven by a large computation. Thus, in addition to possible errors in the validity of used results and algorithms, it is subject to several possible other errors that would not be easily controlled (human coding bug, compiler bug, system bug, processor bug, etc). Unlike other numerical computations for which the notion of certificate permits relatively easy verification, (examples include primality testing with Elliptic curve for example (ECPP), integer factorization, odd perfect number bounds) here the verification has the same cost as the total computation which was used to obtain the result (unlike computations with certificates). It is thus difficult to consider such results as “proved” in a strong sense as a pure mathematical proof. This problematic is expected to be more and more important in the future, as results “computationnaly proved” are likely to be more frequent. Discussion about validity of our RH verification until the 1013-th zero is the object of section 3.3.1.  »

Maintenant, tu voudras bien me donner au moins la référence d'un article publié dans une revue à comité de lecture qui affirme ce que tu dis. La référence à wikipedia n'étant pas suffisante pour cela. Dans l'état actuel de nos connaissances, il me semble peu probable que l'on puisse calculer à un coût pas trop élevé une intégrale complexe pour des valeurs astronomiques sans que l'erreur cumulée dépasse 1.Claudeh5 (d) 24 novembre 2009 à 22:09 (CET)[répondre]

Mmm... Bon, d'accord, je vois ce que tu veux dire. Mais, à ce sens, le théorème des quatre couleurs n'est pas prouvé non plus, si? D'un autre côté, si la situation près du 1013 ème zéro est loin d'être clair, je suppose que tu m'accordera qu'elle l'est pour, mettons, les 106 premiers zéros, non ? En tout état de cause, je crois que, pour le lecteur profane de ces articles, il y a une différence essentielle entre un calcul numérique approché des zéros (mettons par une méthode de Newton, par exemple) qui aboutirait à une valeur de la partie réelle de 1/2 mais à à 10^-50 près, et une méthode de preuve rigoureuse de ce que cette partie réelle =1/2, mais la méthode reposant sur des calculs qui, par leur longueur, présentent en effet un risque (informatique) non négligeable d'^être erronés (d'un autre côté, quand, comme c'est le cas ici, ces calculs ont été refaits plusieurs fois par des équipes indépendantes, le risque est-il toujours si grand?)
Je n'avais pas lu ta dernière intervention. Le papier de Gourdon est évidemment une référence, mais je ne vois pas en quoi ce que je dis en mérite une : je discute d'un algorithme théorique fort simple, c'est les problèmes pratiques de sa mise en œuvre pour de grandes valeurs de T qui se posent. Les remarques de Gourdon semblent plus philosophiques qu'autre chose, dans la mesure où il discute des sources d'erreur possible. Bien évidemment, à la limite, on peut admettre que de toute façon, cela ne donnera jamais une preuve de RH, et inversement, que si on trouvait par cette méthode un zéro hors de la ligne critique, on revivifierait les calculs... En résumé, disons que cela donne un peu confiance (mais les résultats obtenus sur des choses comme les violations des règles de Rosser (et pourquoi pas les oscillations autour de Li(x)) tendent à montrer que finalement, RH est peut-être fausse...), mais que l'intérêt de la chose ne justifie peut-être pas qu'on tente d'obtenir une certitude algorithmique--Dfeldmann (d) 24 novembre 2009 à 22:24 (CET)[répondre]
Je te donne un exemple théorique. Pour calculer les zéros de zêta, on celcule Z(t), fonction réelle à valeurs réelles. A partir de quelle précision sur Z(t) es-tu certain qu'il y a un zéro entre deux valeurs et pas plus ? Qu'il n'y en a pas entre deux valeurs de même signe ?Claudeh5 (d) 24 novembre 2009 à 23:00 (CET)[répondre]
Si je n'ai que cette information sur Z, évidemment. Mais ici, on sait bien plus de choses (par exemple, on peut compter les zéros par la formule des résidus, montrer qu'ils sont simples par minoration de la dérivée de Z, etc.)--Dfeldmann (d) 25 novembre 2009 à 13:17 (CET)[répondre]
Oh, tu en sais des choses ! Plus que les spécialistes qui en sont encore à chercher s'il existe ou non des zéros multiples, donc une minoration de Z'(t). Je te rappelle aussi que l'on connaît que peu de choses sur S(T) qu'on n'est pas capable de majorer mieux que O(ln T)... Donc i te faut intégrer sur un rectangle de largeur 1. Avec quelle formule ? Combien de points ? car il faut contrôler l'erreur de méthode...Claudeh5 (d) 25 novembre 2009 à 13:26 (CET)[répondre]
(Au fait, le premier zéro de zêta est aux environs de 1/2+14 i)
Mais non... Je sais en effet des tas de choses (c'est pas parce que j'avais pas la valeur numérique 1/2 +14,134...i en tête que ça prouve mon incompétence) , mais c'est pas le point, je parle de ce qui est "facile" (les petites valeurs de T, pour lesquels les spécialistes (et pas qu'eux) savent très bien qu'il n'y a pas de zéros multiples), pas des problèmes près du milliardième zéro. J'aimerais bien que tu ne t'arqueboutes pas sur tes positions (et que tu arrêtes de me prendre pour un débutant), mais que tu me donnes par exemple des réponses claires aux questions simples que je pose : 1) est-ce que les trois premiers zéros (ceux étudiées par Riemann) sont, de manière certaine prouvés être sur la ligne critique ? 2) Si ta réponse est oui, pour quelle valeur de N n'es-tu plus sûr que les N premiers zéros y soient: N=1000 ? N=106? N=1013? 3) considères-tu le théorème des 4 couleurs comme mathématiquement prouvé ?--Dfeldmann (d) 25 novembre 2009 à 13:38 (CET)[répondre]
Je vais essayé de répondre à tes questions.
  1. personne ne semble avoir revendiqué le calcul du premier zéro de la fonction zêta de Riemann. Titchmarsh donne quelques éléments sur le calcul des zéros mais cela reste fragmentaire. Pour ce qui est des calculs effectués entre 1903 et 1935, ils ont été faits par des méthodes non automatiques et donc sujets à erreurs. Cependant les calculs de Lehmer (1956) et continuateurs ayant été réalisés par d'autres méthodes, on peut affirmé que les premiers calculs sont justes, encore que je n'ai pas les mémoires correspondants ! A partir des calculs de Rosser & Yohe & Schoenfeld (1969), la question a changé de sens et il faut prendre le problème comme l'a indiqué Gourdon car les 3 500 000 zéros calculés ne peuvent en aucun cas être vérifiés à la main. Ce qu'on peut dire c'est qu'il est quasi certain qu'il n'y a pas d'erreur dans les calculs. Ces calculs se vérifiant par recalculs des précédents.
  2. Je ne sais pas. Je dirai à priori 1041.
  3. Pour ce qui est du théorème des 4 couleurs, la question est toujours posée mais plusieurs autres programmes ont donné des résultats identiques. On cherche encore une preuve mathématique.Claudeh5 (d) 25 novembre 2009 à 15:12 (CET)[répondre]
Je suis bien conscient de tout ça, mais je continue à penser que tu mélanges des choses distinctes (et qui, de fait, sont souvent mal gérées par la communauté des mathématiciens). D'abord, en théorie, des erreurs de calcul (et aussi de raisonnement) sont toujours susceptibles d'avoir échappé à tout le monde, même pour des calculs simples ; c'est seulement en pratique que la vérité de 6*9 =54 (et pas 42), ou celle du théorème de Pythagore n'est pas discutable. La frontière est floue, mais j'ai tendance à penser (et ça, c'est un TI) que le 23ème nombre de Mersenne est bien premier, alors que le 43ème, j'attendrai peut-être une revérification indépendante ou deux. Du coup, les calculs que nous discutons ici, c'est une question de degré : tu penses qu'ils ne sont convainquants que jusqu'au 1000 ème zéro, je pense qu'ils le sont bien plus haut, mais, somme toute, peu importe, puisque seul un contre-exemple (qui, lui, demanderait vérification, revérification, voire rerevérification) serait essentiel ; on sait que, du moins sur des questions proches, le premier contre-exemple pourrait parfaitement être du côté du 10300 ème zéro, et que ça, aucun calcul direct ne le montrera jamais ; pour que la situation change, il faudrait que des arguments (au moins heuristiques) montrent qu'une vérification jusqu'à N suffit pour assurer une forte confiance dans l'hypothèse, et cela, bien sûr, on en est bien loin.
Inversement, je pense que la confiance dans ce genre de calculs est quand même encadrée par des tas de choses : des estimations théoriques précises de l'erreur commise, tant sur les valeurs de la fonction zeta (ou sa dérivée) que lors d'un calcul numérique, et d'autres estimations (nettement moins théoriques) sur les risques d'erreurs informatiques (tant matérielles que logicielles) ; il me semble que Gourdon, par exemple, malgré ses réserves, ne se contente pas de dire : "Bon, j'ai fait faire le calcul par telles machines, avec tel algorithme ; voilà le résultat, pensez-en ce que vous voudrez", mais donne de nombreuses analyses des sources d'erreur éventuelles...--Dfeldmann (d) 25 novembre 2009 à 16:06 (CET)[répondre]

Question 7[modifier le code]

Ma lecture du sujet est un peu incomplète,donc je n'ai peut-être pas vu cette question.A-t-on un résultat plus faible que l'hypothèse de Riemann,c'est-à-dire:Dans l'ensemble des zéros non triviaux de la fonction,la probabilité pour qu'un zéro soit hors de l'axe 1/2 est égale à zéro.--90.15.72.18 (d) 26 novembre 2009 à 09:01 (CET)[répondre]

Je crains bien que cette phrase, telle quelle, n'ait que peu de sens ; quelle est, plus généralement, le sens de "la probabilité que A soit vraie (ou que A soit démontrable) est p"? Si vous voulez parler d'un résultat tel que "on connait explicitement un ensemble de mesure (relative) nulle contenant tous les zéros hors de la droite critique" ou "la proportion de zéros hors de la droite critique par rapport à ceux qui y sont, de parie imaginaire <T, tend vers zéro avec T tendant vers l'infini", la réponse est "on ne sait pas", voir les passages pertinents dans les résultats du type "zones sans zéros"...--Dfeldmann (d) 26 novembre 2009 à 09:52 (CET)[répondre]
La réponse est oui: cela porte le nom d'hypothèse de densité. On sait que la proportion par rapport à N(T) de ces zéros éventuels tend vers 0 quant on s'écarte de 1/2 et que T tend vers l'infini. Voir http://fr.wikipedia.org/wiki/Histoire_de_la_fonction_z%C3%AAta_de_Riemann#L.27hypoth.C3.A8se_de_densit.C3.A9 pour plus de précision.Claudeh5 (d) 26 novembre 2009 à 10:44 (CET)[répondre]
Ce qui ne signifie pas qu'il ne peut exister une suite infinie de zéros dont la partie réelle tendrait vers 1 (par exemple).Claudeh5 (d) 26 novembre 2009 à 10:55 (CET)[répondre]
Dans le paragraphe "hypothèse de densité",vous avez écrit que :d'après le résultat,on a vu que la proportion N(T,s) des zéros de partie réelle supérieure à s par rapport à N(T)tend vers 0 quand T tend vers l'infini.De quel résultat parlez-vous ?
Dans le paragraphe "combien de zéros sur l'axe 1/2, il y a ce résultat (qui pourrait être ailleurs ...)

"A l'inverse, un théorème de Bohr et Landau de 1914 montre que la croissance de | ζ(s) |^2 est liée à la répartition des zéros. La valeur moyenne de | ζ(s) | 2 est majorée sur les droites σ = cte. On en déduit que la proportion des zéros en dehors de la bande 1 / 2 − δ < σ < 1 / 2 + δ tend vers 0 quand T tend vers l'infini. Cela mène tout droit à l'hypothèse de densité."

Si je comprends bien l'état du problème, l'hypothèse de densité n'est pas démontrée pour tout s dans [1/2,1].--90.0.147.56 (d) 26 novembre 2009 à 16:05 (CET)[répondre]
Effectivement, elle n'est pas démontrée. Pas avec l'exposant 2 mais avec l'exposant 12/5=2,4 . Donc, compte tenu de la valeur de N(T), c'est mieux seulement pour (1-s)<5/12 donc s>7/12.Claudeh5 (d) 26 novembre 2009 à 16:23 (CET)[répondre]
Un point de détail dans le paragraphe hypothèse de densité,de Histoire de la fonction ;Vous dites : il existe une constante telle que pour tout ,etc...Est-ce il existe C telle que pour tout ,ou pour tout il existe C.Dans le premier cas C n'est pas .Un passage à la limite peut ne pas être le même dans les deux cas.--90.14.208.66 (d) 27 novembre 2009 à 21:36 (CET)[répondre]
Oui, c'est vrai. Il faut lire "D'un résultat de Ingham (1940) et de Huxley (1972), sans hypothèse, pour tout ε > 0, on montre l'existence d'une constante C = C(ε) telle que, on ait".Claudeh5 (d) 28 novembre 2009 à 08:48 (CET)[répondre]
Dans ce cas,si on veut passer à la limite pour tendant vers 0,il faut que les constantes soient majorées.Je suppose que c'est le cas,et pour toutes les autres majorations de N(s,T)!
je n'ai pas l'expression de mais si cette constante était bornée quand , on n'aurait pas de dans la formule du tout. Donc tend vers l'infini avec .Claudeh5 (d) 1 décembre 2009 à 21:40 (CET)[répondre]
Je reviens sur la question d'ensemble. Si on ne sait pas calculer exactement l'intégrale de contour sur le rectangle [1/2+d;1+iT] d'une fonction dont les pôles sont les zéros de ou prouver que cette intégrale est nulle,on ne pourra jamais démontrer l'hypothèse de Riemann par cette voie.
Exact. D'ailleurs on n'a pas réussi à le faire. Disons clairement les choses: on a des expressions exactes mais qui ne sont pas manipulables pour en tirer une preuve.Claudeh5 (d) 1 décembre 2009 à 21:40 (CET)[répondre]
Quel est l'intérêt d'avoir les majorations de N(s,T) développées dans l'hypothèse de densité,puisqu'on savait déjà,d'après la citation sur un théorème de Bohr et Landau,que la proportion des zéros en dehors de l'axe 1/2 tend vers 0 quand T tend vers l'infini? D'autant plus que les majorations donnent une mesure maximum de la décroissance,mais elle est très lente,et encore sur une partie de la bande ]1/2,1].--90.0.191.250 (d) 30 novembre 2009 à 00:29 (CET)[répondre]
L'intérêt est de diminuer le plus possible le nombre des zéros en dehors de l'axe critique. Tout progrès est bon à prendre. et permet par contre coup de nouvelles estimations sur la valeurs de zeta dans la bande critique. Si par exemple on démontre que l'on ne peut pas avoir , alors l'hypothèse de Riemann est fausse ! Tout se tient et tout progrès peut avoir une influence décisive dans cette affaire.Claudeh5 (d) 1 décembre 2009 à 21:40 (CET)[répondre]
Les majorations de l'hypothèse de densité sont assez "minimes" numériquement.D'autant plus que tend vers l'infini quand tend vers 0,donc les majorations sont intéressantes si ne décroit pas trop.

Question 8[modifier le code]

Est-ce qu'on a démontré que tous les zéros de dans la bande [0,1] sont simples? Ou plus précisément,qu'aucun zéro de ne peut être un zéro de ?--90.0.190.169 (d) 5 décembre 2009 à 01:21 (CET)[répondre]

Non, même en admettant l'hypothèse de Riemann. Voir l'article histoire de la fonction zêta de Riemann. Il y a un paragraphe sur la simplicité des zéros avec les derniers résultats.Claudeh5 (d) 5 décembre 2009 à 07:40 (CET)[répondre]
J'ai été amené à m'interroger sur la simplicité des zéros,parce que je pense que ce problème "ressemble" un peu à celui de l'existence de zéros en dehors de l'axe 1/2.Je pense (peut-être que je m'avance un peu trop) que si on pouvait démontrer que tous les zéros sont simples,on aurait une idée de la façon dont il faudrait procéder pour démontrer la non existence de zéros hors de l'axe 1/2.--90.0.190.169 (d) 5 décembre 2009 à 19:52 (CET)[répondre]
En l'état actuelle de nos connaissances, l'hypothèse de simplicité des zéros n'est pas démontrée, pas même sous l'hypothèse de Riemann. Par contre on peut démontrer l'hypothèse de Riemann et l'hypothèse de simplicité par la conjecture de Mertens affaiblie. Mais la conjecture de Mertens forte est fausse et la conjecture de Mertens généralisée est considérée comme fausse aussi. Quant à démontrer l'hypothèse de Riemann en supposant seulement l'hypothèse de simplicité, personne n'a réussi à le faire.Claudeh5 (d) 5 décembre 2009 à 21:52 (CET)[répondre]
1)Demande de confirmation

La série qui définit la fonction peut être dérivée terme à terme,si je ne me trompe pas.En effet,sa convergence vient du critère de sommation d'Abel,et la majoration du critère est uniforme sur tout demi-plan ,donc la convergence est uniforme sur ce demi-plan,et on peut dériver la série terme à terme, que ce soit la dérivée holomorphe,ou les dérivées partielles en ou en t. Par ailleurs,les zéros de dans la bande ]0,1[ sont les mêmes que ceux de ,et c'est aussi le cas des zéros doubles.

oui, parfaitement. eta n'a des zéros en plus que sur Re(s)=1 par rapport à zeta.

2)Une question

Du point de vue des conséquences,quelles sont les différences entre l'hypothèse de Riemann stricte,et une situation où on trouverait des zéros hors de l'axe 1/2,mais un nombre infime ? (par exemple un sur 10^6,ou un sur 10^9)--90.15.74.18 (d) 15 décembre 2009 à 18:18 (CET)[répondre]

Elles sont énormes. Si l'on a un zéro de partie réelle b>1/2, la série de Dirichlet 1/zeta ne converge que sur Re(s)>b. Le reste dans le théorème des nombres premiers est de la forme x^b \ln x cad que l'on a pi(x)=li(x)+Omega(x^b ln(x)) ce qui signifie que le reste est vraiment de cet ordre et non d'un ordre strictement plus petit: il existe une suite de x tendant vers l'infini tels que ... Les théorèmes battis sur la véracité de l'hypothèse de Riemann ou d'une des hypothèses de Riemann généralisées sont faux ou du moins, on doit refaire une démonstration... La conjecture de Legendre est fausse ainsi que pleins d'autres propriétés (test de primalité, ...)Ce qui compte ainsi c'est le Sup des parties réelles des zéros de zeta.Claudeh5 (d) 15 décembre 2009 à 22:18 (CET)[répondre]

Lien avec l'électron[modifier le code]

Le lien entre la fonction zeta de Riemann et les électrons ajouté par un contributeur sous IP ne semble pas être un TI http://scholar.google.fr/scholar?hl=fr&q=function+zeta+riemann+electron Je laisse à plus calé que moi le soin de l'introduire éventuellement dans l'article.---- El Caro bla 2 janvier 2010 à 20:31 (CET)[répondre]

a voir, bien que je ne sois pas un spécialiste de la question de l'électron.Claudeh5 (d) 3 janvier 2010 à 07:09 (CET)[répondre]
Non, c'est pas si simple (et la façon dont l'IP en parlait était particulièrement douteuse). Mais en fait, faut le relire, l'article  : il mentionne explicitement cette histoire (les travaux de Berry), de façon bien plus rigoureuse, ici (... et chaos quantique) ; le fait que ce soit les états de l'électron n'a aucun intérêt particulier, ce qui compte, c'est l'idée du Hamiltonien --Dfeldmann (d) 3 janvier 2010 à 07:15 (CET)[répondre]
oui, c'est tout à fait mon impression concernant l'IP. Par contre j'ai commencé à regarder un article de Harney, The stability of electron orbital shells based on a model of the Riemann-zeta function dont le résumé est "It is shown that the atomic number Z is prime at the beginning of the each s1, p1, d1, and f1 energy levels of electrons, with some fluctuation in the actinide and lanthanide series".Claudeh5 (d) 3 janvier 2010 à 07:56 (CET)[répondre]
Mieux vaut en rire : vu que les nombres en questions sont des nombres premiers de l'intervalle 1-100 (avec des fluctuations...), la probabilité d'une coïncidence est déjà >0,1, sans parler des arbitraires là dedans, et de ce que les considérations sur les carrés (et les doubles de carrés) ramènent la question sur Z à se demander pourquoi 5, 17 et 37 sont premiers... Disons qu'avec cette approche (au moins ainsi résumée), je ne croit pas que le prix Clay craigne grand chose.--Dfeldmann (d) 3 janvier 2010 à 08:26 (CET)[répondre]
A ça, c'est vrai. Bon, je crois qu'on peut conclure: on ne change rien. J'ai déjà lu d'autres liens qui ne mènent à rien de sérieux. Par contre j'ai trouvé un curieux texte en espagnol [1] sur la fonction zeta de Riemann qui affirme des choses que j'ignorais ... mais que je crois fausses ! il est dit "En 1912, Littlewood [24] prob´o que la conjetura de Riemann implica la de Lindel¨of. El rec´ıproco es falso." ? La réciproque est fausse en quel sens ? fausse au sens que l'hypothèse de Lindelöf n'implique pas l'hypothèse de Riemann ou fausse au sens que l'on n'a pas démontré que l'hypothèse de Lindelöf implique l'hypothèse de Riemann ? A ma connaissance on est dans la seconde signification.Claudeh5 (d) 3 janvier 2010 à 10:43 (CET)[répondre]
  1. Calderon, La Funci'on Zeta de Riemann, Rev. Real Academia de Ciencias. Zaragoza. 57: 67–87, (2002)

Question des zéros doubles[modifier le code]

Je me suis demandé si on ne pouvait pas considérer la question des zéros doubles de la fonction (donc de ) d'un point de vue géométrique,et par conséquent s'il n'y avait pas eu des recherches dans cette direction.Je m'explique.Si on appelle R(s),I(s),A(s),B(s) respectivement la partie réelle de ,sa partie imaginaire,la dérivée partielle de la partie réelle en , et la dérivée partielle de la partie réelle en t,on a quatre fonctions définies sur la partie ]0,1[ (au moins) du plan (,t) à valeur dans ,qui définissent 4 surfaces dans l'espace.Les zéros d'une de ces fonctions sont la trace de la surface correspondante dans le plan (,t).Ces traces ne sont pas des variétés au sens classique,car elles ne peuvent pas être paramétrées sur un intervalle réel.Ce sont des réunions (infinies) de composantes connexes qui séparent la bande ]0,1[ en régions où la fonction est >0, et régions où elle est négative.

Or les zéros doubles de sont des points où les 4 traces doivent être concourantes.Il semble tout à fait logique que les traces deux à deux aient des points d'intersection (si on prend le cas de R et I,ce sont bien sûr les zéros de situés sur l'axe 1/2).Il semble déjà beaucoup moins probable qu'existent des points où 3 des traces soient concourantes,et il semble encore bien moins probable des points où les 4 traces soient concourantes.Ceci reste intuitif sous cette forme.Je me suis demandé si on n'avait pas des résultats sur ces "probabilités". Mais est-ce possible d'avoir des résultats généraux,sans tenir compte de la forme des fonctions elles-mêmes ?--90.0.57.94 (d) 6 janvier 2010 à 02:22 (CET)[répondre]

D'une part les zéros de la fonction eta de Dirichlet sur l'axe 1 sont simples, d'autre part les zéros de eta sont exactement ceux de zeta en dehors de l'axe 1. Donc cela n'apporte rien de considérer eta à la place de zeta.
On ne sait sur la question de la multiplicité des zéros que ce qui est dit dans la paragraphe sur la multiplicité. L'hypothèse de Riemann n'exclue pas, semble-t-il, qu'il existe des zéros doubles. On sait seulement que les hypothèses de Mertens implique la simplicité des zéros. Enfin, l'étude de la fonction zeta de Riemann ne fait pas intervenir de probabilité. Claudeh5 (d) 6 janvier 2010 à 08:08 (CET)[répondre]
Les résultats de l'hypothèse de densité que vous avez donnés dans l'Historique, donnent quand même des limites supérieures de probabilité d'avoir des zéros de tels que > ou<.Rien n'interdit de définir des probabilités de certains événements sur des ensembles.Mais sinon,vous pouvez dire proportion relative d'occurrence d'un certain événement sur un ensemble,non?
a quand même une forme plus simple que dans la bande ]0,1[.La partie réelle de est

Ses dérivées ont aussi des formes simples,par exemple la dérivée en

Si on veut prouver certains résultats numériquement,c'est plus simple d'utiliser ,au lieu de .--90.0.180.10 (d) 6 janvier 2010 à 09:21 (CET)[répondre]

N'auriez vous pas oublié le facteur (1-2^(1-s)) ?Claudeh5 (d) 6 janvier 2010 à 10:31 (CET)[répondre]
Non,c'est bien ,sa partie réelle et les dérivées partielles de celle-ci.Le facteur apparait dans qui est donc plus compliquée.--90.0.71.164 (d) 12 janvier 2010 à 09:19 (CET)[répondre]
Dès que vous voudrez utiliser ces calculs (lesquels ?) sur eta pour les traduire sur zêta, il apparaitra le facteur (1-2^(1-s)). Dès que vous ferez un calcul, il vous faudra majorer le reste qui, lui, ne se majore pas par le premier terme négligé. Je ne vois pas ce que l'on gagne là-dedans.Claudeh5 (d) 12 janvier 2010 à 10:08 (CET)[répondre]

Petite erreur ?[modifier le code]

Je me demande si dans le prolongement de dzeta avec la formule d'Euler Mac Laurin, on n'applique pas la formule à x->x^s au lieu de s->x^s, à vérifier... Florent H., 31 Mars 2010

exact, je corrige--Cbigorgne (d) 31 mars 2010 à 18:25 (CEST)[répondre]

Discussion (Question)[modifier le code]

Quel est le comportement de la série pour et ,où est le nème nombre premier ?

On sait que la série de la fonction de Riemann en ne converge pas quand (on peut le montrer en montrant que le critère de Cauchy est infirmé,sur la partie réelle par exemple).

Je pense qu'on a la même conclusion pour la série en ,mais le calcul est plus complexe car il faut une bonne évaluation de (provenant du théorème des nombres premiers).Le problème est évoqué dans l'article Histoire de la fonction zêta de Riemann,mais uniquement le cas où ,qui est beaucoup plus simple.--90.0.51.43 (d) 20 avril 2010 à 09:56 (CEST)[répondre]

La théorie des séries de Dirichlet précise que l'ensemble des valeurs s pour lesquelles il y a convergence simple est un demi-plan éventuellement vide. La question que vous posée se traite de manière élémentaire: la série des 1/pn est divergente ainsi qu'il a été démontré par Euler (il s'agit du cas sigma=1, non de sigma=0, dans Histoire de la fonction zêta de Riemann). La divergence étant obtenue pour s=sigma=1, la série est divergente pour toute autre valeur dans le demi-plan Re(s)<1.Claudeh5 (d) 22 avril 2010 à 08:31 (CEST)[répondre]
Le calcul n'est d'ailleurs pas compliqué: il suffit de développer le produit eulérien pour s>1 et de faire tendre s vers 1+.Claudeh5 (d) 22 avril 2010 à 21:59 (CEST)[répondre]

Arnaque pour la démo de la relation fonctionnelle[modifier le code]

Dans la démo proposée de la relation entre dzeta(z) et dzeta(1-z) tout est bon jusqu'à la décomposition en Fourier de {u}-{2u} mais ensuite cà coïnce. Même si l'échange somme/intégrale est licite (il ne l'est sûrement pas) la ligne suivant est fausse : l'intégrale de 0 à l'infini se décompose en différence de deux intégrales absolument convergentes ; le changement 2u=v donne que cette différence est (2^s-1) \int_0^infiny u^{-1-s} sin(2n \pi u) du. De là un simple changement de variable permet de trouver un équivalent en l'infini de la forme constante * n^s. Or la série des n^{-1+s} ne converge pas ... — Le message qui précède, non signé, a été déposé par un utilisateur sous l’IP 81.220.66.45 (discuter), le 26 juillet 2010 à 03:41.

la variable s est dans ]0,1[ donc positive. Donc -1 - s < -1; de ce fait la série n^{-1-s} converge.Claudeh5 (d) 26 juillet 2010 à 14:31
Il s'agit de la série des n^{-1+s} et non n^{-1-s} — Le message qui précède, non signé, a été déposé par un utilisateur sous l’IP 81.220.66.45 (discuter), le 27 juillet 2010 à 12:32.
Le document source est http://arxiv.org/abs/math/0305191 et on y lit à la fin de la première page "where the last equation is valid at first when −1 < <s < 0, but it obviously provides the analytic extension to the whole plane.".Claudeh5 (d) 27 juillet 2010 à 14:27
Ok, il s'agit en fait de la subtilité suivante : la formule
(2^s-1) dzeta(s)/s= \int_0^\infty x^{-s-1}( {x} - {2x} ) dx
est démontrée pour $0< Re(s) < 1$ mais se prolonge à $-1< Re(s) < 1$ du fait de l'oscillation de {x} - {2x} (l'intégrale est alors impropre en l'infini). — Le message qui précède, non signé, a été déposé par un utilisateur sous l’IP 81.220.66.45 (discuter), le 9 août 2010 à 14:06.

Une incorrection (au moins) dans la démonstration de l'expression intégrale de contour[modifier le code]

La démonstration faite par le contributeur qui a voulu prouver l'expression de par une intégrale de contour est tout à fait contestable.En effet,si je reprends le passage où il écrit:

qu'est-ce qui m'empêche d'écrire

 ?

auquel cas les intégrales et se neutralisent,et on va trouver que l'intégrale de contour globale est nulle quand !

Cependant,il y a là une erreur,mais où est l'erreur?Si on peut aboutir à une telle erreur,c'est qu'il y a une incorrection dans sa démonstration.--90.15.75.38 (d) 9 août 2010 à 23:51 (CEST)[répondre]

En fait, il y a non pas une entourloupe mais une incorrection à dire que l'on prend des nombres d'argument 2π . On doit considérer une droite paramétrée reliant le bas de la demie circonférence décrite par le paramètre ν et le point à l'infini. Ces points sont "presque" d'argument 2π mais la différence (non nulle) tend vers 0 avec ν. Cela empêche de pouvoir écrire .Claudeh5 (d) 28 octobre 2010 à 20:07 (CEST)[répondre]

Critique de la démonstration sous dérouler dans #la bande critique et l'hypothèse de Riemann[modifier le code]

Dans la démonstration,on écrit "Or, ξ(s) est réelle pour t=0 et également pour , de sorte que la variation totale autour du rectangle est 2 fois la variation autour de la moitié en partant de s=2 ".OK pour la prémisse,mais la conclusion ?

Si est réelle,l'argument est constant (égal à avec un certain k).Donc sa variation entre 1/2+iT et 2,en passant par 1/2,n'est-elle pas nulle ?

Et deuxième question : comment peut-on prendre l'intégrale de entre 1/2+iT et 1/2,sachant que la fonction n'est pas holomorphe sur le segment à cause des zéros 0<t<T (qui sont des pôles pour cette fonction s'ils sont simples).Est-ce que les zéros et les pôles,dans le principe de l'argument,ne sont pas intérieurs à la boucle de l'intégration ?

--90.0.178.173 (d) 18 novembre 2011 à 15:23 (CET)[répondre]

avertissement : modif. cosmétiques[modifier le code]

bonjour. Je vais faire des modif. cosmétiques en vue d'une éventuelle proposition de bon article. Peut-être aussi que je soulèverai des points à améliorer dans le fond. Ghalloun [allôôô?] 6 juillet 2012 à 13:32 (CEST)[répondre]

j'ai commencé un travail de relecture de ce magnifique article. Je suis arrivé aux relations fonctionnelles, non incluses. Je vous livre les points qui m'ont paru à améliorer éventuellement :
  • dans la définition par série de Dirichlet, que se passe-t-il sur la droite Re(z) = 1, privée de 1 ? Est-ce important à dire? Est-ce évident? Il s'agit, je crois, d'appliquer la fonction exp dans C, et mes souvenirs sont loin...
  • toujours dans définition par série de Dirichlet, j'ai l'impression qu'il y a une ambiguïté entre série de Dirichlet et de Riemann. Le titre parle de Dirichlet, l'article détaillé de Riemann. Ca ne fait pas très sérieux. A moins d'être au carré sur les différences entre les deux (domaine de déf. réel ou complexe?), on s'embrouille.  OK
  • dans liens avec les fonctions arithmétiques, on a à un endroit une majoration par K / log(x), dont on déduit une convergence. Est-elle simple, normale, absolue? Doit-on le dire ou bien c'est évident?
La convergence est simple et n'est pas absolue. La source ne précise pas si la convergence serait uniforme. Il semblerait d'après ce qui est dit plus loin dans un paragraphe sur la fonction 1/zeta qu'il y aurait aussi convergence sur la droite Re(s)=1 toute entière mais ce n'est pas clair pour moi. --Cbigorgne (d) 7 juillet 2012 à 14:45 (CEST)[répondre]
  • pourquoi ne pas regrouper les sections 2 à 6 dans une grande section qui pourrait s'intituler : premières considération sur zêta, ou étude de zêta pour des valeurs complexes de partie réelle supérieure à 1 ? Ca structurerait mieux l'article, je pense.  OK
Bonne idée.--Cbigorgne (d) 7 juillet 2012 à 14:45 (CEST)[répondre]
  • extension à C - {1}. J'ai cru déceler une erreur dans l'expression du lacet. A vous de vérifier. le bout de lacet circulaire donne un "grand O". Ne pourrait-on pas justifier ce résultat? Il tombe un peu comme un oiseau tout cuit. J'ai essayé de le retrouver, ça n'a rien d'évident.
Peut-être que vous faites référence au sujet de la discussion ci-dessus (#Une incorrection (au moins) dans la démonstration de l'expression intégrale de contour) sur une ambiguïté sur la description du lacet. Cela ne semble pas avoir été corrigé.--Cbigorgne (d) 7 juillet 2012 à 14:45 (CEST)[répondre]
  • sommation d'Abel, le domaine de s devrait apparaître avant la première formule. Là, tout est implicite, alors que l'enjeu est important. Non?
Je suis d'accord pour expliciter le domaine de s.--Cbigorgne (d) 7 juillet 2012 à 14:45 (CEST)[répondre]
  • formule de Ramaswami. Ne pourrait-on pas exprimer la somme de façon extensive avec un signe somme, comme pour la première?  OK
Je suis d'accord pour expliciter la formule.--Cbigorgne (d) 7 juillet 2012 à 14:45 (CEST)[répondre]
  • Développement de Laurent. Je vois une expression fonctionnelle gamma (s-1). N'est-ce pas plutôt une constante indicée, comme celle décrite plus haut?  Non. C'est le produit de deux facteurs : gamma x (s-1). Au temps pour moi.
merci pour vos éclairages. Et en route pour le BA! Ghalloun [allôôô?] 7 juillet 2012 à 14:12 (CEST)[répondre]

Avancement[modifier le code]

J'ai lu les critères pour un avancement A, et il me paraît qu'il manque juste la garantie d'une structure de l'article adéquate. Ne pourrait-on pas regrouper dans relation fonctionnelle les deux sections suivantes? Ghalloun [allôôô?] 8 juillet 2012 à 20:56 (CEST)[répondre]

Bonjour, c'est plus clair avec ce regroupement. Cordialement.--Cbigorgne (d) 10 juillet 2012 à 14:04 (CEST)[répondre]
Merci Cbigorgne! Je vais continuer ma relecture. Ghalloun [allôôô?] 10 juillet 2012 à 15:22 (CEST)[répondre]

Série de Riemann et série de Dirichlet[modifier le code]

Bonjour! On fait référence parfois dans l'article à série de Riemann et à série de Dirichlet. Peut-être que cela pourrait donner lieu, à un endroit ou à un autre, à une note indiquant la différence entre les deux types de séries? Si c'est véritablement la même chose, il faudrait plutôt alors fusionner l'article Riemann dans l'article Dirichlet, plus complet. Je n'ai pas lu assez en détail ces deux articles pour savoir s'il y a ou non différence ce concept. Ghalloun [allôôô?] 11 juillet 2012 à 13:49 (CEST)[répondre]

En fait, ça ne semble pas nécessaire : les séries de Riemann sont un cas particulier des séries de Dirichlet avec un numérateur qui vaut 1. Ghalloun [allôôô?] 17 juillet 2012 à 14:21 (CEST)[répondre]

Parties réelles[modifier le code]

Je ne voudrai pas être critique mais franchement je trouve les "nouvelles" parties réelles TRḔS moches ! Ce n'est pas assez noir et trop petit ! Claudeh5 (d) 21 juillet 2012 à 21:51 (CEST)

Pourquoi faudrait-il que ce soit plus noir ? Préfèrerais-tu Re(s) ? Quant à "trop petit", le but est justement (à la fois pour l'accessibilité et la lisibilité) d'éviter au maximum LaTeX dans le corps du texte. Qu'en pensent les autres ? Anne (d) 21 juillet 2012 à 22:04 (CEST) P.S. Je viens de découvrir ça : (s), ça te (vous) plait ?[répondre]

Personnellement, et vu l'usage général fait au moins en France de Re(s), je préfère avec un e comme ça: e(s).Claudeh5 (d) 22 juillet 2012 à 19:44 (CEST)[répondre]

Quant à moi, je préfère aussi le "R" stylisé, avec ou sans "e". Ghalloun [allôôô?] 22 juillet 2012 à 20:43 (CEST)[répondre]
J'avais fait abstraction de mes goûts et essayé d'uniformiser selon la typo majoritaire dans cet article. À mon tour de trouver « moche » ce mélange de 2 polices (encore pire pour m), mais si je suis la seule alors tant pis. Je peux me charger de transformer tout (en texte et en LaTeX), mais avant, est-ce que les contributeurs qui s'intéressent actuellement à cet article pourraient "voter" ? Plutôt qu'une liste exhaustive de combinaisons, je propose divers critères (si j'en oublie, rajoutez), puis mes préférences :
  1. Fraktur : a=non, b=oui x=indifférent
  2. Minuscule complémentaire : a=non, b=oui x=indifférent
  3. modèle math : a=pas du tout, b=juste sur l'opérateur, c=à la fois sur l'opérateur et sur la variable x=indifférent
  4. balises big : a=non, b=oui x=indifférent
  5. caractères gras : a=non, b=oui x=indifférent
Je préfère Re(s) = 1a+2b+3b+4a+5a (3c est plus joli que 3b mais plus contraignant : si on veut uniformiser le rendu, on est alors "obligés" de mettre ce modèle aussi quand la variable apparaît seule dans la même phrase)
Anne (d) 22 juillet 2012 à 21:21 (CEST)[répondre]
Il y a de plus un très léger problème avec les titres... Et je ne sais pas si le fonte Fraktur a les minuscules. Donc je réponds normand... MAIS toujours avec deux lettres Re ou Im (plus lisibles) et pas une seule lettre. réponses aux questions : 1x+2b+3x+4x+5b Claudeh5 (d) 22 juillet 2012 à 22:42 (CEST)[répondre]
Ok, alors je vais mettre partout (en LaTeX) et Re(s) (en mode texte) ou Re(s) mais seulement si tu insistes, car ce gras me capte un peu trop l'œil et donne un rendu différent de LaTeX.
Pour le Fraktur, si je comprends bien tu n'y tiens pas mordicus à cause des problèmes de police dans les titres et de minuscule de fonte différente. Je suis d'accord et j'espère que Ghalloun aussi.
Je fais très attention à ne pas introduire de coquilles (et j'en élimine, comme toi, à la fois de nouvelles qui sont de mon fait et de très anciennes) mais ça peut à nouveau m'arriver hélas. Anne (d) 25 juillet 2012 à 12:52 (CEST)[répondre]

Quelqu'un a-t-il une démonstration courte du fait que ln3/ln2 est irrationnel ?Claudeh5 (d) 22 juillet 2012 à 19:51 (CEST)[répondre]

Idée comme ça : si ln3/ln2 = p/q, alors 3^q = 2^p, or 2 et 3 sont premiers entre eux, donc c'est absurde. J'ai bon, ou pas ? Kelam (mmh ? o_ô) 22 juillet 2012 à 20:09 (CEST)[répondre]
Oui, ça me semble juste. Ghalloun [allôôô?] 22 juillet 2012 à 20:47 (CEST)[répondre]

Je l'avais déjà rajoutée (et généralisée) à 17h58, via ce lien ln 3ln 2 se trouvant irrationnel. Anne (d) 22 juillet 2012 à 21:18 (CEST)[répondre]

excellent! si ln3/ln2 = p/q, alors 3^q = 2^p donc notamment 3^q est congru à 0 (mod 2). Or 3 étant congru à 1 mod 2, on a 3^q congru à 1^q =1 mod 2 ce qui rend la congruence 3^q =0 (mod 2) impossible. Je crois que le lien sur nombre irrationnel n'explique pas grand chose (et manque d'exemple(s) en plus !).Claudeh5 (d) 22 juillet 2012 à 22:24 (CEST)[répondre]
Le lien est directement sur la section adhoc de cet article et la démo m'y semble à la fois plus générale et plus simple, mais on peut la détailler là-bas...Anne (d) 22 juillet 2012 à 22:33 (CEST)[répondre]
Plus générale, oui: «Dès que deux entiers strictement supérieurs à 1 n'ont pas le même radical, autrement dit le même ensemble de facteurs premiers, il ne peuvent avoir de puissance (d'exposant entier non nul) commune, donc le quotient de leurs logarithmes est irrationnel. Le théorème de Gelfond-Schneider permet alors d'en déduire que ce quotient est même transcendant. Par exemple : ln 3⁄ln 2 est transcendant»; par contre plus simple (avec le théorème de Gelfond-Schneider, j'en doute). De plus la formulation initiale est plus un argument d'autorité (croyez moi !) qu'une démonstration. Je détaille. D'autant que le vocabulaire n'est pas adapté (radical) à un lecteur 'ordinaire'.Claudeh5 (d) 22 juillet 2012 à 23:06 (CEST)[répondre]
J'y suis repassée entre-temps (Désolée, ça a probablement occasionné un conflit d'édit) mais n'hésite(z) pas à améliorer encore.
Gelfond-Schneider ne fait évidemment pas partie de la réponse à ta question mais vient en prime : peut-être vaut-il mieux mettre les exemples avant ?
"Radical" n'est pas indispensable donc je l'ai placé entre parenthèses, mais laissé pour la culture.
Anne (d) 22 juillet 2012 à 23:21 (CEST)[répondre]
J'ai vu que tu avais encore amélioré plus tard. Je maintiens que c'était non seulement plus général mais plus simple (que les congruences) mais mon texte initial était effectivement très mal rédigé. Anne (d) 25 juillet 2012 à 12:52 (CEST)[répondre]
le problème essentiel est le "donc" dans ta démonstration. Quant à dire que c'est plus général, j'en doute fort:

Soient m et n deux nombres entiers n'ayant pas les mêmes nombres premiers et soit p un nombre premier apparaissant dans la décomposition de m mais pas dans celle de n (il y en a au moins un, quitte à inverser m et n). Si ln m/ln n était rationnel alors il existerait deux entiers a et b tels que ln m/ln n=a/b donc, en prenant l'exponentielle de chaque côté, on aurait m^b =n^a. L'équation donne forcément que n^a est congru à 0 modulo p puisque m est congru à 0 (mod p). Ceci est impossible car n n'est pas congru à 0 (mod p) sinon il serait multiple de p. Donc n^a, quelque soit a entier, n'est jamais congru à 0 (mod p). Donc ni a ni b ne peuvent exister: le nombre ln m/ln n est irrationnel.Claudeh5 (d) 25 juillet 2012 à 15:19 (CEST)[répondre]

ok, ta nouvelle démo n'est plus "moins générale". La mienne est identique sans les lourdeurs inutiles des congruences. Anne (d) 25 juillet 2012 à 15:41 (CEST)[répondre]

Deuxième formule de Perron[modifier le code]

Dans Fonction zêta de Riemann#Que devient la série de Riemann sur l'axe Re(s) = 1 ? je lis : « Pour les séries de Dirichlet de ζ'/ζ, ln ζ et 1/ζ, l'application de la deuxième formule de Perron montre que les deux dernières séries convergent sur l'axe 1 en dehors de s = 1 tandis que la première ne converge pas sur l'axe 1. » La preuve de cette « application » me passe au-dessus de la tête (elle pourrait peut-être être effleurée comme exemple dans l'article sur la formule de Perron ?) mais j'aimerais bien être sûre au moins de comprendre la fin de cet énoncé : est-ce que ça veut dire que la série de ζ'/ζ diverge en tous les points de l'axe ?Anne (d) 25 juillet 2012 à 12:52 (CEST)[répondre]

Bonjour Anne, d'après Tenenbaum, édition de 2008, page 252, exercice 279, la série de Dirichlet de ζ'/ζ diverge en tous les points de l'axe. Cordialement.--Cbigorgne (d) 25 juillet 2012 à 14:00 (CEST)[répondre]
diverge en un point s_0 signifie essentiellement "ne converge pas"! elle peut donc diverger vers l'infini ou osciller indéfiniment ... J'ai montrer un exemple de ce dernier cas justement dans ce paragraphe.Claudeh5 (d) 25 juillet 2012 à 14:27 (CEST)[répondre]
@Claude : je sais. @Cbigorgne : merci. Anne (d) 25 juillet 2012 à 15:08 (CEST)[répondre]

Vous en pensez quoi[modifier le code]

de ce que j'ai ajouté ? Claudeh5 (d) 26 juillet 2012 à 14:40 (CEST)[répondre]

  • D'abord merci : je t'ai "piqué" 2 idées pour améliorer l'analogue dans Série de Dirichlet#Propriétés analytiques (l'idée d'écrire directement la série de 0 à l'infini au lieu de tergiverser comme Petkov-Yger, et l'idée de calculer les ζ(-k)).
  • Ton calcul des ζ(-k) peut être grandement simplifié (vois là-bas ce que j'en ai fait)
  • ζ(-k) hors-sujet dans cette section, à caser plutôt dans les 2 sous-sections correspondantes de la section « Propriétés diverses de la fonction »
  • Ta preuve de (utilisé aussi juste après, dans « Par une intégrale de contour » où elle figurait il y a pas longtemps) doublonne avec celle de « Calcul d'intégrales ». (Je l'y avais déplacée récemment, quand je rédigeais « Série de Dirichlet#Propriétés analytiques » avec dans la tête la section que tu viens d'écrire.)
Anne (d) 26 juillet 2012 à 16:47 (CEST)[répondre]
Mon but initial était de faire un sort (mauvais) à ça:

«De la définition de la fonction zêta par une intégrale de contour[1], on déduit que pour tout entier naturel n, ζ(–n) est le rationnel suivant :

Bn + 1 est un nombre de Bernoulli.» qui est une sottise historique (puisque la valeur aux entiers négatifs a été découverte par Euler qui ne connaissait pas les "intégrales de contour"). Je n'avais pas vu que le calcul était doublonné dans les applications au calcul d'intégrales. Bien vu pour les séries de Dirichlet.Claudeh5 (d) 26 juillet 2012 à 17:47 (CEST)[répondre]

  1. Tenenbaum 2008, p. 233-234
Fait. Anne (d) 30 juillet 2012 à 14:18 (CEST)[répondre]

seconde partie de relecture[modifier le code]

Bonjour! J'ai continué à relire l'article. Voici quelques questions, remarques et pistes d'améliorations :

  1. dans l'introduction de ln (zêta), je lis zêta plus grande que 1. Ne faut-il pas dire "plus grande que 0"?
  2. dans la 5e ligne sur la fonction de Mertens, "on sait qu'elle est également valable pour ..." Je ne sait pas à quoi "elle" renvoie. Peut-on l'expliciter?
  3. sur l'axe Re(s)=1, on fait référence au théorème de Dirichlet. J'ai cherché ce qu'il énonce, et Wikipedia en indique au moins trois... Par ailleurs, on ne sait pas d'où sort l'espèce de formule d'Euler-Mal Laurin qui vient rapidement. Je pense que ce serait bien de l'indiquer.
  4. relation fonctionnelle approchée : à quoi sert le paramètre 'h' dans les inégalités?
  5. théorie de la fonction Mu. estimation "d'après Kolesnik". Il faudrait donner une référence, je pense. Ghalloun [allôôô?] 28 juillet 2012 à 20:16 (CEST)[répondre]
  1. Je suppose qu'il s'agit de la phrase "La fonction ζ étant réelle sur l'axe réel et plus grande que 1, le logarithme de cette valeur existe et est réel. ". La fonction zeta est définie par 1+des termes qui sont tous positifs pour s réel >1. Donc zeta est plus grande que 1. En fait zeta est décroissante sur ]1, infini[ passant de +infini à 1 qui est valeur asymptotique. Il n'y a rien à corriger.
  2. La formule visée est
    qui est valable pour Re(s)>1 de manière évidente mais également valable pour s=1+it, t non nul.
  3. le théorème de Dirichlet énonce que zeta admet un pôle en 1 de résidu 1. autrement dit zeta(s)= 1/(s-1) + fonction entière. Quant à la formule d'Euler-Mac Laurin, il s'agit d'une formule très générale et très importante qui lie les intégrales et les séries permettant soit d'évaluer les séries par une intégrale et des termes supplémentaires soit inversement d'évaluer une intégrale par une série et des termes supplémentaires. Evidemment, si la formule est théorique, on peut aussi l'utiliser pratiquement quand on sait calculer l'intégrale ou la série.
  4. h est une valeurs arbitraire mais non nulle qui empêche que l'on s'approche trop près de 0.
  5. Kolesnik, On the order of ζ(1/2+it) and Δ(R), Pacific Journal of mathematics, T98,N°1, 1982.Claudeh5 (d) 28 juillet 2012 à 23:21 (CEST)[répondre]

Inégalités[modifier le code]

J'ai mis un bandeau "section à sourcer" sur la section 4.7 "Inégalités". Il est vrai que l'inégalité de La Vallée Poussin est classique et qu'on la trouve facilement un peu partout. Mais ça n'est pas le cas des deux autres. Laforgia et Natalini ont publié plusieurs articles ensemble. Des 4 autres auteurs cités, les 3 derniers ne sont jamais répertoriés ensemble dans MathSciNet; et Ahsani n'existe carrément pas. (De plus: est-il vraiment nécessaire de mentionner les deux inégalités, la deuxième impliquant la première?). Sapphorain (discuter) 4 juillet 2014 à 10:50 (CEST)[répondre]

O de Landau: pas "calligraphique"[modifier le code]

En théorie des nombres, dans la littérature, les symboles O de Landau ne sont jamais écrit avec un O "calligraphique". Il semble que cette mode nous soit venue de certains traités d'analyse d'erreur en informatique. En tout cas dans un article de théorie des nombres ça ne se justifie pas (et ça fait mal aux yeux). J'ai donc enlevé tous les \mathcal devant O. Sapphorain (discuter) 8 juillet 2014 à 10:21 (CEST)[répondre]

Historique au début?[modifier le code]

Y a-t-il une raison pour que la section "Historique" se trouve presque à la fin (et très peu visible dans la table des matières)? Normalement une telle section se trouve tout au début. Je pense qu'il faudrait la déplacer au début, et la renommer "Histoire", ou même "Histoire de la fonction zêta de Riemann" comme l'article auquel on renvoie. Sapphorain (discuter) 8 juillet 2014 à 22:54 (CEST)[répondre]

Affirmations non sourcées concernant Euler, Hadamard, La Vallée Poussin[modifier le code]

J'ai supprimé la contribution de Utilisateur:Dhallewyn, qui n'est pas sourcée et contient des affirmations fausses ou imprécises. (1) Euler n'a jamais conjecturé l'équation fonctionnelle; il a réussi à calculer des valeurs de zêta pour des arguments entiers négatifs, ce qui a semble-t-il inspiré Riemann. En tous les cas il faut une référence. (2) Ça n'est ni Hadamard, ni La Vallée Poussin qui a montré l'équivalence du théorème des nombres premiers avec la non annulation de zeta sur Re(s)=1. Si mes souvenirs sont bons, c'est von Mangoldt ou Landau, quelques années après la preuve du TNP. Là aussi, il faut une référence. Sapphorain (discuter) 12 juillet 2014 à 23:43 (CEST)[répondre]

REPONSE à Utilisateur:Sapphorain . D'après (mais pas seulement) un article de Michel Balazard (CNRS umr 2615) "Un siècle et demi de recherches sur l'hypothèse de Riemann" Il dit mot pour mot la phrase suivante au sujet de Euler : " Enfin il découvre l'équation fonctionnelle de zêta la démontrant quand s est entier, et la considérant comme très plausible pour tout s réel (présentation en 1749,publication en 1768) " .. Maintenant en ce qui concerne TNP, vous avez probablement raison, il s'agit soit de la preuve de Hadamard après une remarque de Landau OU une preuve de Hadamard avant une remarque de Landan ( c'est bien Landau après vérification (cf.Colmez))Il faudrait vérifier les dates. Merci d'avoir modifier (supprimer), j'avoue avoir été imprécis (ce qui n'est pas permis) Dhallewyn (discuter) 14 juillet 2014 à 19:40

(1) Si Michel Balazard l'écrit, ça doit être vrai et c'est moi qui me trompe. Il faut cependant une référence précise à l'article d'Euler. (2) Concernant l'équivalence du TNP avec zeta(s)≠0 pour Re(s)=1, il semble en effet que ça soit dû à Landau. Mais de nouveau, il faut une référence (à l'article de Landau, et non pas à un article de quelqu'un d'autre qui affirme sans référence que c'est dû à Landau!). J'ai cherché dans les oeuvres complètes de Landau, et pour l'instant je n'ai rien trouvé. Cordialement. Sapphorain (discuter) 14 juillet 2014 à 23:10 (CEST)[répondre]

Reponse :: Dhallewyn (discuter 18 juillet 2014 à 03:52 Voilà un lien sur l'article de Landau je crois : http://www.ams.org/journals/bull/1914-20-07/S0002-9904-1914-02502-9/S0002-9904-1914-02502-9.pdf et, mais cela peut être douteux : http://images.math.cnrs.fr/Jacques-Hadamard-et-le-theoreme.html

Le premier lien est sur le résumé de Gronwall (1914) du Handbuch de Landau (1909), le deuxième sur un article de vulgarisation; aucun n'apporte de réponse à la question posée. Sapphorain (discuter) 19 juillet 2014 à 16:58 (CEST)[répondre]

Dhallewyn (discuter 05 août 2014 à 22:45 En ce qui concerne cela, j'ai trouvé quelque chose, tout d'abord pour confirmer la conjecture d'Euler voir Colmez ( élements d'analyse, d'algèbre et de théorie des nombres).. Puis pour TNP, voir un article de WIKIPEDIA " théorème des nombres premiers" dans la rubrique ébauche de démonstration.

Pour Euler, la référence que j'ai donné à série divergente (Leonhard Euler, « Remarques sur un beau rapport entre les séries des puissances tant directes que réciproques », Mémoires de l'Académie des sciences de Berlin, 17, 1768, p. 83-106 ; Opera Omnia : Series 1, vol. 15, p. 70-90 ; Euler Archive : E352 (lire en ligne) ; ce texte, écrit en 1749, constitue une des premières utilisations de séries clairement divergentes, et, concernant ces formules, il écrit lui-même (p.2) : "cela doit paraître bien paradoxe".) est très explicite (page 1, où il annonce le résultat pour s entier, et le conjecture dans le cas général)--Dfeldmann (discuter) 5 août 2014 à 23:59 (CEST)[répondre]

Bonjour,

J'invite les contributeurs de cet article à déterminer si la présence du Portail:Physique est pertinente ou non. Bien cordialement — t a r u s¡Dímelo! 15 août 2014 à 19:03 (CEST)[répondre]

Serait-il pertinent d'apposer le Portail:Botanique sur l'article Semoir ? Le tout est de savoir si on parle de l'outillage ou du matériel (au sens du langage scientifique). Le semoir fait partie de l'outillage ; la graine qu'il sème fait partie du matériel (végétal, en l'occurrence).
Bon alors, OK, dans ce cas du semoir, le vrai portail est celui de l'agriculture et l'agronomie. Pourquoi ? Parce qu'un semoir ne sert que comme moyen de production dans ce secteur d'activité.
Est-ce que la fonction zêta a été inventée (ou découverte, comme on voudra), spécialement en vue de résoudre des questions liées à des théories physiques (comme le chaos quantique, dont Riemann n'avait pas entendu parler) ? Non.
Cordialement Michel421 (d) 1 août 2015 à 23:39 (CEST)[répondre]

Notification Starus, Cbigorgne, Anne Bauval, Dfeldmann, Sapphorain, Vivarés, Kelam et Valvino : Si cette fonction peut avoir un rôle d'outil en physique (cf le § en dessous), je ne vois pas d'évidence que le portail serve ici, autrement que comme lien interne ; des liens vers un ou deux articles précis seraient de toute façon plus fonctionnels qu'un portail ; pouvez-vous donner votre avis là-dessus ? Cordialement, Michel421 (d) 24 août 2015 à 19:26 (CEST)[répondre]

Je ne vois pas trop l'intérêt de mettre un Portail:Physique à cet article, ou alors il faudrait mettre un tel portail à la quasi totalité des articles de mathématiques.Theon (discuter) 12 octobre 2015 à 10:37 (CEST)[répondre]

OK - j'enlève le portail ✔️ Michel421 (d) 12 octobre 2015 à 18:43 (CEST)[répondre]

Utilité de la fonction zêta dans la théorie du chaos quantique[modifier le code]

Peut-on considérer ceci comme une source valable à mettre éventuellement dans l'article ?

Nombres premiers et chaos quantique

Merci de vos commentaires éventuels Michel421 (d) 19 mai 2015 à 19:39 (CEST)[répondre]

Réduction de l'hypothèse de Riemann à une partie de la bande critique.[modifier le code]

Je crois avoir lu que l'hypothèse de Riemann avait été réduite à une partie de la bande critique, a>=1/2 ou a<=1/2, je ne sais plus, qqn aurait-il plus d'infos svp?

Oui, mais ça n'est pas très récent... Par l'équation fonctionnelle dans la section 4.2 de l'article (Riemann 1859) il "suffit" de montrer qu'il n'y a pas de zéro dans une des deux moitiés de la bande critique, à gauche ou à droite de l'abscisse 1/2. Sapphorain (discuter) 3 août 2016 à 11:12 (CEST)[répondre]

Image du demi-plan Re(s) > 1[modifier le code]

Notification Cbigorgne : Quelle est-elle ? Mais surtout : est-elle incluse dans C\R ? Voire même dans Re(z) > 0, i.e. la série du § sur log(ζ) vérifie-t-elle |Im(D(s))| < π/2 ? Anne (discuter) 4 mars 2017 à 22:03 (CET)[répondre]

A propos de la relation fonctionnelle[modifier le code]

Bonjour, Vous dites que la relation fonctionnelle liant zeta(s) à zeta(1-s) est valable pour tout complexe s distincts de 0 et 1. Cependant pour s = 3; 4; 5;... la fonction s --> Gamma(1-s) devient infini en ces valeurs. Alain Houé90.32.29.2 (discuter) 11 juillet 2017 à 10:39 (CEST)[répondre]

Bonjour, je pense qu'il faut prendre l'égalité comme une égalité entre fonctions méromorphes sur l'ensemble de nombres complexes différents de 0 et de 1. Donc lorsque la fonction s |--> Gamma (1-s) a un pôle, celui-ci est annulé par le facteur Zeta (1-s) ou le facteur sin(πs/2) qui a un zéro du même ordre que le pôle. Cordialement.-- Cbigorgne (discuter) 11 juillet 2017 à 10:56 (CEST)[répondre]
Une rédaction correcte serait de dire que « le terme de droite dans la relation fonctionnelle définit une fonction méromorphe (en dehors de 0 et de 1) qui n'a aucun pôle et égale le premier terme de l'équation Zeta(s). »-- Cbigorgne (discuter) 11 juillet 2017 à 11:02 (CEST)[répondre]

Ordre de présentation inverse dans "Quotients de fonctions zêta par ζ(s)" ?[modifier le code]

Les dernières contributions de Cbigorne dans la section "Quotients de fonctions zêta par ζ(s’')" me laissent perplexe. Elles donnent d’abord la série de Dirichlet pour \zeta’(s)/\zeta(s), sans dire comment on l’obtient, et en déduisent par intégration terme à terme celle pour log\zeta(s). Il me semble qu’en général on fait précisément l’inverse: en utilisant le fait que \zeta(s) ne s’annule pas sur \sigma>1, et que donc la détermination principale du log\zeta(s) est holomorphe dans cette région, on peut calculer le log du produit eulérien pour \zeta(s) à l’aide de la série de Taylor pour log(1-x), et obtenir d'abord la série pour log\zeta(s). Ensuite en dérivant terme à terme on obtient celle pour \zeta’(s)/\zeta(s). Je ne sais pas obtenir directement cette dernière série. Evidemment, s'il est possible de l'obtenir directement, on peut laisser comme ça (peut-être avec une petite explication). Sapphorain (discuter) 24 juillet 2017 à 00:24 (CEST)[répondre]

J'ai supprimé ces contributions, qui se trouvent déjà, dans un ordre logique, dans la section « Définition de ln\zeta et de sa dérivée ». Sapphorain (discuter) 26 juillet 2017 à 00:11 (CEST)[répondre]

Pour quelle raison les valeurs impaires ne sont-elles pas toutes irrationnelles ?[modifier le code]

Bonjour, en regardant cette formule :

... ne peut-on pas en conclure que toutes les valeurs impaires positives, sauf 1, sont irrationnelles ? Avec ce produit eulérien, et du fait de l'infinité des nombres premiers, il en découle que toutes les valeurs impaires, comme les paires, ne peuvent pas s'écrire sous la forme d'un rationnel ?

Athanatophobos 25 janvier 2020, 11:00 CET.

Bonjour Athanatophobos ÉmoticôneUn produit infini de rationnels peut parfaitement être rationnel. Par exemple, pour rester dans le sujet de l'article, on a la jolie formule:
(laissée en exercice au lecteur)
--Dfeldmann (discuter) 25 janvier 2020 à 12:29 (CET)[répondre]
Merci, je comprends : le fait que zeta(2) vaut
, montre, du fait de l’irrationalité de pi, qu'il y a une infinité de nombres premiers. Mais la réciproque est fausse comme le montre votre exemple.

Athanatophobos 26 janvier 2020, 9:00 CET

Bonjour Athanatophobos ÉmoticôneOui ; cela dit, c'est bien trop compliqué ; il faut plutôt remarquer (Euler) que le produit eulérien correspondant à s=1 () diverge (comme la série harmonique); on en déduit que la série des inverses des nombres premiers diverge aussi (ce qui est un résultat plus fort).--Dfeldmann (discuter) 26 janvier 2020 à 12:39 (CET)[répondre]